GENERAL PRACTITIONER EXAM
Question Summary
0 of 100 questions completed
Questions:
- 1
- 2
- 3
- 4
- 5
- 6
- 7
- 8
- 9
- 10
- 11
- 12
- 13
- 14
- 15
- 16
- 17
- 18
- 19
- 20
- 21
- 22
- 23
- 24
- 25
- 26
- 27
- 28
- 29
- 30
- 31
- 32
- 33
- 34
- 35
- 36
- 37
- 38
- 39
- 40
- 41
- 42
- 43
- 44
- 45
- 46
- 47
- 48
- 49
- 50
- 51
- 52
- 53
- 54
- 55
- 56
- 57
- 58
- 59
- 60
- 61
- 62
- 63
- 64
- 65
- 66
- 67
- 68
- 69
- 70
- 71
- 72
- 73
- 74
- 75
- 76
- 77
- 78
- 79
- 80
- 81
- 82
- 83
- 84
- 85
- 86
- 87
- 88
- 89
- 90
- 91
- 92
- 93
- 94
- 95
- 96
- 97
- 98
- 99
- 100
Information
Hi, Welcome to General Practioner Exam
You have already completed the Exam before. Hence you can not start it again.
Exam is loading...
You must sign in or sign up to start the Exam.
You have to finish following Exam, to start this Exam:
Results
0 of 100 questions answered correctly
Your time:
Time has elapsed
You have reached 0 of 0 points, (0)
Average score |
|
Your score |
|
Categories
- Not categorized 0%
Pos. | Name | Entered on | Points | Result |
---|---|---|---|---|
Table is loading | ||||
No data available | ||||
- 1
- 2
- 3
- 4
- 5
- 6
- 7
- 8
- 9
- 10
- 11
- 12
- 13
- 14
- 15
- 16
- 17
- 18
- 19
- 20
- 21
- 22
- 23
- 24
- 25
- 26
- 27
- 28
- 29
- 30
- 31
- 32
- 33
- 34
- 35
- 36
- 37
- 38
- 39
- 40
- 41
- 42
- 43
- 44
- 45
- 46
- 47
- 48
- 49
- 50
- 51
- 52
- 53
- 54
- 55
- 56
- 57
- 58
- 59
- 60
- 61
- 62
- 63
- 64
- 65
- 66
- 67
- 68
- 69
- 70
- 71
- 72
- 73
- 74
- 75
- 76
- 77
- 78
- 79
- 80
- 81
- 82
- 83
- 84
- 85
- 86
- 87
- 88
- 89
- 90
- 91
- 92
- 93
- 94
- 95
- 96
- 97
- 98
- 99
- 100
- Unanswered
- Answered
- Review
-
Question 1 of 100
1. Question
1 pointsA colon cancer and five surrounding lymph nodes are biopsied. Histopathology shows that the lesion extends into the muscularis propria with two positive lymph nodes. What is the Aster Coller modification of Duke´s staging system?
Correct
Incorrect
Explanation:
The Aster Coller modification of Duke´s staging system is commonly used to stage colon carcinoma. Stage A is when the tumor does not extend into the muscularis. Stage B1 is when the tumor extends into the muscularis without lymph node involvement, and stage C1 is stage B1 with lymph node involvement. Stage B2 is when the tumor extends past the muscularis without lymph node involvement, and stage C2 is stage B2 with lymph node involvement. Stage D is distant metastatic disease.
-
Question 2 of 100
2. Question
1 pointsA woman aged 82 years is found unresponsive following a minor surgical procedure. What should be the initial intervention
Correct
Incorrect
Explanation:
Interventions of unresponsive patients should always follow the ABCs of basic life support. An unresponsive state most likely occurs secondary to lack of oxygen to the brain or excess carbon dioxide. The first step in managing an unresponsive patient is to secure the airway and provide oxygenation. Only then does fluid resuscitation begin. Diagnostic studies follow once the initial life support measures are instituted.
-
Question 3 of 100
3. Question
1 pointsA 38 year old female presents to her physician with a breast lump noted during self examination. Out of the following, which is NOT necessary in her evaluation?
Correct
Incorrect
Explanation:
Obtaining a series of skeletal radiographs in the initial evaluation of an individual with a newly discovered breast lump is not useful in the elucidation of the diagnosis. Mammography is very important to identify the size and presence of calcification in the mass. A physical examination by the physician is essential to independently document the lesion and to measure it. It is necessary to obtain a family history, since women who have first degree female relatives with breast cancer are at greater risk themselves. Determining the medications the patient has taken recently is necessary, since some, such as hormones, can be associated with breast lesions.
-
Question 4 of 100
4. Question
1 pointsA male aged 56 years with a prosthetic heart valve needs to undergo urgent abdominal surgery to rule out an intra abdominal abscess. The patient is taking warfarin and his current prothrombin time is 21. The most appropriate course of action is
Correct
Incorrect
Explanation:
In the case of a patient in need of urgent surgery, fresh frozen plasma is the fastest and safest way to correct the prothrombin time. Non urgent surgery should be postponed, but urgent surgery, such as in this case where an abscess is suspected, should not be delayed. Protamine reverses heparin, although it has no effect on prothrombin time. Vitamin K will counteract the effect of coumadin, but it takes several days to work, and it makes anticoagulation after surgery extremely difficult. Packed red blood cell transfusion will not correct an abnormal prothrombin time
-
Question 5 of 100
5. Question
1 pointsWhat is the primary treatment for colonic carcinoma?
Correct
Incorrect
Explanation:
While radiation and chemotherapy are appropriate as adjuvant treatment of colonic carcinoma, they are inadequate alone. Local excision is also inadequate. Wide resection with drainage of regional lymphatics is the treatment of choice.
-
Question 6 of 100
6. Question
1 pointsA male aged 55 years is brought to the emergency room because of presumed Sterno ingestion. Which one of the following condition is he likely to develop?
Correct
Incorrect
Explanation:
The major toxicity derived from Sterno is due to its methanol content. Methanol is metabolized in the body to formaldehyde and formic acid, which can damage the retina. The other symptoms listed are not side effects specific to Sterno.
-
Question 7 of 100
7. Question
1 pointsRegarding bariatric surgery the correct statement is which one of the following?
Correct
Incorrect
Explanation:
Bariatric surgery is a major gastrointestinal procedure. Bariatric surgery in adolescents raises social, psychological and developmental issues, but adolescents are not excluded from surgery, and some hospitals have specialised programmes for younger patients. Potential candidates for surgery are those with a body mass index (BMI) exceeding 40, or BMI > 35 with serious co-morbidities (eg sleep apnoea, type 2 diabetes). Post-operative mortality ranges from 0.1-2 %. Vomiting is a risk associated with bariatric surgery, as is dumping syndrome and nutritional deficiencies. There is no evidence as yet; that bariatric surgery reduces cardiovascular mortality in patients.
-
Question 8 of 100
8. Question
1 pointsA 56 year old woman has a 2 month history of severe diarrhea. The stools are loose without blood or mucous. There is severe halitosis that began along with diarrhea. Subtotal gastrectomy was done at age 38 for severe peptic ulcer disease. It included a Billroth II antecolic gastrojejunostomy. 2 years ago she had an episode of hematemesis, and was found to have a marginal ulcer at the anastomotic site. She declined transthoracic vagotomy and opted for medical management. The likely diagnosis is
Correct
Incorrect
Explanation:
The history is classical, with severe diarrhea, weight loss, and halitosis as a result of fecal contamination of the gastric contents. The setting for this fistula to develop is also well described in the vignette: an antecolic anastomosis that develops a marginal ulcer that is not properly treated. The ulcer eventually eats its way into the adjacent colon.
Diarrhea occurs in both the afferent loop syndrome (A) and the dumping syndrome (B) but in both cases the complication develops shortly after the original surgery. Neither of these problems would first show up nearly 20 years after surgery. Gastric cancer (D) can indeed develop in the remaining stomach after peptic ulcer surgery, and it happens a good many years later thus fitting the pattern of this vignette. But the manifestations would not include diarrhea and halitosis, and the history would not include the marginal ulcer as in this case. Gastrinoma (E) if responsible for the original ulcer would have led to continued peptic ulcer disease right away, not many years after the operation. The diarrhea of Zollinger Ellison is watery, and it occurs along with other manifestations of peptic ulcer disease, such as pain, peroration, or bleeding. -
Question 9 of 100
9. Question
1 pointsA 57 year old diabetic male is admitted with a diagnosis of epididymo-orchitis. WBC count was 22,000/mL He was started on IV ciprofloxacin, scrotal elevation, and scrotal ice packs. On third day there is persistent right hemi scrotum edema with scrotal wall erythema. The testicle is tender to palpation. Blood and urine cultures are negative. Ultrasound shows an increase in regional blood flow of the right epididymis and confirms the thickened edematous scrotal wall and it also shows complex, loculated fluid collection within the right hemi scrotum. The most appropriate management at this time is
Correct
Incorrect
Explanation:
This patient has an epididymo-orchitis that is complicated by the development of a scrotal abscess. The appropriate treatment of an abscess anywhere in the body is drainage. The technique of drainage can vary, depending on the location of the abscess with a scrotal abscess, exploration and open drainage is the most appropriate. The location is easy to gain access to, the actual procedure is generally well tolerated, and this is the only technique that ensures all the purulent fluid is drained completely. Aspiration (A) carries the risk of not removing all the fluid; therefore, there is a risk for recurrence. Aspiration of an abscess should be reserved for collections that would require major surgical intervention to reach. A change in this patient´s antibiotics will not treat him completely (B). Management of a patient with presumptive epididymo-orchitis on admission with intravenous antibiotics is appropriate. Most patients with this diagnosis improve with antibiotics when a patient´s condition does not respond appropriately, however, it is necessary to investigate other possible diagnoses. As stated, this patient requires surgical drainage.
This patient has not improved from a clinical or laboratory standpoint. His ultrasound shows an abscess; therefore, continuing with the current regimen would be inappropriate (C). Fournier gangrene (D) is defined as a necrotizing fasciitis that presents as a progressive, fulminating infection of the male genitalia, initially, the patient may present with only induration of the penis or scrotal skin. Erythema and crepitus develops and quickly extends over the perineum and abdominal wall. Treatment requires immediate wide surgical debridement of the involved skin and subcutaneous tissue. -
Question 10 of 100
10. Question
1 pointsA 32 year old diabetic man develops fever, confusion, and severe abdominal pain 10 hours after fistulectomy. He vomits 5 times and passes 3 loose stools. He has Crohn´s disease for several years. He is currently on metformin, mesalamine, and corticosteroids. On exam today, his temperature is 38.9oC, BP is 100/52 mm Hg, pulse is 122/min, and respirations are 18/min. CBC shows lymophocytosis with eosinophilia. CXR is normal. Serum Na+ is decreased, K+ is increased and glucose is 60 mg/dL. The likely cause for his condition is
Correct
Incorrect
Explanation:
Administration of long term glucocorticoids suppresses the hypothalamic pituitary adrenal axis (CRH) by exerting a negative feedback control to suppress corticotropin-releasing harness (CRH) secretion and, consequently, adrenocorticotropic hormone (ACTH) secretion, which leads to tertiary adrenal insufficiency. The atrophic adrenal glands become incapable of producing the high quantity of steroids needed during times of stress, such as surgery. The major clinical manifestation of adrenal crisis is shock, but patients may also have nonspecific symptoms such as anorexia, nausea, vomiting, abdominal pain, diarrhea, lethargy, fever, confusion, or coma. Laboratory studies of this patient reveal hyponatremia, hyperkalemia due to mineralocorticoid deficiency, and hypoglycemia due to glucocorticoid deficiency. Other findings include lymphocytosis and eosinophilia. Adrenal crisis could have been prevented in this patient by increasing steroid doses on the day of the surgery and then tapering off to usual levels a few days after surgery. Adrenal hemorrhage may also cause an adrenal crisis. Causes include septicemia induced Waterhouse-Friderichsen syndrome (fulminant meningococcemia) or the over-anticoagulation of patients. Waterhouse Friderichsen syndrome is usually much more acute in onset and often diagnosed at autopsy. Moreover, this patient has a history of chronic steroid use, making tertiary adrenal insufficiency the most likely cause.
-
Question 11 of 100
11. Question
1 pointsA 46 year old female has bone pain, myalgias, constipation, and a depressed mood. She has been hospitalized twice in the last year for urolithiasis. Labs reveal hypercalcemia and elevated PTH level. Her renal function is normal, and neck ultrasound reveals a parathyroid mass. Which laboratory abnormality would be expected in this patient?
Correct
Incorrect
Explanation:
The patient described is suffering from primary hyperthyroidism. The most common cause of primary hyperthyroidism is a parathyroid adenoma; parathyroid carcinoma and hyperplasia are much less common. This condition most commonly occurs in women over the age of 40 years as a solitary adenoma. Genetic syndromes such as MEN I, IIa, and IIb may also be associated with parathyroid adenomas, among other findings. Typical presenting complaints are those associated with hypercalcemia. These include recurrent urolithiasis, constipation, myalgias, bone pain, polyuria, fatigue, and psychologic changes. Patients may be asymptomatic at presentation, however, with the diagnosis made following discovery of an elevated calcium level on routine blood tests. Laboratory studies typically reveal an inappropriately elevated parathyroid hormone (PTH) level, hypercalcemia, hypophosphatemia, and elevated urine calcium. Chronically elevated PTH levels can cause defective bone mineralization and remodeling, leading to osteitis fibrosa cystica, a condition characterized by the formation of “brown tumors” in bone, particularly in the jaw. In cases in which the bone is significantly affected, the serum alkaline phosphatase is elevated, not decreased (E). Hypernatremia (A) typically occurs in the setting of excessive free water loss and is not associated with primary hyperparathyroidism. Hyperphosphatemia (B) and low levels of dihydroxy (24, 25(OH) 2) vitamin D occur in the setting of renal failure/renal osteodystrophy, one cause of secondary hyperparathyroidism. Hyponatremia (C) can occur in the syndrome of inappropriate antidiuretic hormone (SIADH), which is a paraneoplastic condition associated with small cell carcinoma of the lung. Another paraneoplastic syndrome associated with this malignancy is hypercalcemia caused by production of PTH related peptide by the malignant cells, resulting in hypercalcemia and low PTH levels. -
Question 12 of 100
12. Question
1 pointsA previously healthy 48 year old woman has a sudden onset of severe back and leg pain 2 hours ago when attempting to lift heavy object. She says that she felt “a bolt of lightening” running down the back of her leg, and she still has very severe pain that prevents her from walking or moving. It is aggravated by coughing, sneezing, or straining. After confirming the diagnosis the appropriate treatment is
Correct
Incorrect
Explanation:
The clinical features are those of a herniated lumbar disc. The diagnosis should be confirmed with an MRI, and then the patient should be treated conservatively with short term bed rest (approximately a few days to a couple of weeks) and analgesics. Most patients get better with this simple approach. Giving antibiotics (B) assumes an infectious process. Infections can occur in the lumbar spine or the discs, but their symptoms do not start suddenly, like this vignette describes. A body cast (C) might be needed for fractures, scoliosis, or other spinal pathology, but casting is not needed for an extruded disc. Radiotherapy (D) assumes a neoplastic process. Although a weakened bone may indeed rupture suddenly, such patients are usually known to have had the kind of tumor that is likely to metastasize to bone (in women, breast cancer would lead the list), and would have been complaining of localized bony pain before the process gets to the point of fracture. Surgical decompression (E) would have been required if she had sphincteric deficits or perineal anesthesia.
-
Question 13 of 100
13. Question
1 pointsA 75 year old complains of persistent soiling of his underwear. 10 weeks ago underwent surgery for his anus problem. He was in a lot of pain when he went into the hospital. Today his underwear has some fecal soiling. There is a scar on the left side of his anus and a small opening within this scar. There is granulation tissue at this opening. An indurate cord is palpated from the scar to the anus and when this cord is pushed, a small amount of fluid is expressed from the opening lateral to the anus. The most likely diagnosis is
Correct
Incorrect
Explanation:
This patient is suffering from fistula-in-ano. This is a chronic form of per anal abscess that is spontaneously or surgically drained, but in which the abscess cavity, does not heal completely. The cavity becomes an inflammatory track with a primary opening (internal opening) in the anal crypt at the dentate line and a secondary opening (external opening) in the perianal skin. They are classified according to the relation of the fistula to the sphincter muscle. Most of these patients have a history of anorectic abscess with intermittent drainage with fecal soiling and occasional perinea discomfort. The external opening is usually visible red elevation of granulation tissue with purulent or serosanguineous drainage on compression. Necrotic and draining tumor must be ruled out (with proctoscopy or sigmoidoscopy) and treatment is byway of fistulotomy to undo of the Fistula, eliminate the primary opening, and establish adequate drainage. An anal fissure (A) typically occurs in young women who have exquisite tenderness with defecation and blood streaks that cover the stool. Patients with an ischiorectal abscess (C) are febrile and complain of exquisite per rectal pain that does not allow them to sit down or have a bowel movement Examination is classic for an abscess (rumor, dolor, color, and tumor) that is lateral to the anus. Incision and drainage is the treatment of choice. A pilonidal sinus (D) is caused by infection of a hair follicle in the sacrococcygeal area. They may present as an acute abscess at the sacrococcygeal area that ruptures spontaneously, leaving an unhealed sinus with chronic drainage. If no abscess is present, then examination will reveal a pit or pits in the midline that represent infected hair follicles. Squamous cell carcinoma (E) of the anus is seen in HIV and homosexual men with receptive sexual practices. On examination, there is a fungating mass that grows out of the anus and metastatic inguinal nodes may be palpated.
-
Question 14 of 100
14. Question
1 pointsA 74 year old female with atrial fibrillation, hypertension and osteoarthritis has bright red blood per rectum. Her medications include warfarin, nifedipine, metoprolol, and ibuprofen. First incident occurred 12 hours ago with 3 subsequent episodes. Her vital signs are stable. Her rectal exam is notable for heme-positive stool. Labs reveal serum potassium of 3.1mEq/L and creatinine of 1.4. The management of this patient is
Correct
Incorrect
Explanation:
The patient has a lower GI bleed, the differential of which includes colon cancer, diverticulosis, arteriovenous malformation, and hemorrhoids. The etiology is not initially s important as stabilizing the patient and preventing further bleeding. A blood transfusion would be first and foremost but is not an option. Reversing the coagulopathy, which is likely due to non- monitored warfarin use, is the next most crucial step. This can best be accomplished using fresh frozen plasma in the acute setting. It is always prudent to monitor elderly patients, especially those with a cardiac history, for volume overload when transfusing FFP; this is because it involves transfusing a large volume of fluid in a short period of time. Elderly patients should be diuresed, if necessary. Factor VII (A) has been approved for use in bleeding, but its limited testing in clinical trials and high expense limit its use to critical patients, typically in disseminated intravascular coagulopathy (DIC) in the operating room. It is used more in obstetric and gynecologic treatment than anywhere else.
Vitamin K (B) is always indicated in patients with an abnormal INR, and is particularly useful in coagulopathy due to nutritional Vitamin K deficiencies and liver disease. However, its effect is significantly less potent and slower than FFP, with no effect for about l2 hours. Therefore, it should also be given in this patient but it is the most critical agent to use in reversing her ongoing bleed. -
Question 15 of 100
15. Question
1 pointsA 74 year old female with atrial fibrillation, hypertension and osteoarthritis has bright red blood per rectum. Her medications include warfarin, nifedipine, metoprolol, and ibuprofen. First incident occurred 12 hours ago with 3 subsequent episodes. Her vital signs are stable. Her rectal exam is notable for heme-positive stool. Labs reveal serum potassium of 3.1mEq/L and creatinine of 1.4. The management of this patient is
Correct
Incorrect
Explanation:
The patient has a lower GI bleed, the differential of which includes colon cancer, diverticulosis, arteriovenous malformation, and hemorrhoids. The etiology is not initially s important as stabilizing the patient and preventing further bleeding. A blood transfusion would be first and foremost but is not an option. Reversing the coagulopathy, which is likely due to non- monitored warfarin use, is the next most crucial step. This can best be accomplished using fresh frozen plasma in the acute setting. It is always prudent to monitor elderly patients, especially those with a cardiac history, for volume overload when transfusing FFP; this is because it involves transfusing a large volume of fluid in a short period of time. Elderly patients should be diuresed, if necessary. Factor VII (A) has been approved for use in bleeding, but its limited testing in clinical trials and high expense limit its use to critical patients, typically in disseminated intravascular coagulopathy (DIC) in the operating room. It is used more in obstetric and gynecologic treatment than anywhere else.
Vitamin K (B) is always indicated in patients with an abnormal INR, and is particularly useful in coagulopathy due to nutritional Vitamin K deficiencies and liver disease. However, its effect is significantly less potent and slower than FFP, with no effect for about l2 hours. Therefore, it should also be given in this patient but it is the most critical agent to use in reversing her ongoing bleed. -
Question 16 of 100
16. Question
1 pointsA young married couple has been trying to conceive for the past 1 year and has been unsuccessful. The wife is deemed to be fertile after examination. Husband´s systemic review and exam is also normal. After finding a decreased sperm count, normal ejaculate volume, and abnormal morphology, the most appropriate next step in management is
Correct
Incorrect
Explanation:
Male factors account for infertility in approximately one third of cases (one third are female factors, one third are combined). Evaluation is warranted if no pregnancy occurs after 12 months of unprotected intercourse, which is the standard definition of in fertility. The physical examination should look for any features of hypogonadism and an evaluation of testicular size. The First laboratory step in male fertility analysis is semen analysis. If semen analysis reveals normal count, morphology, and volume, the next step should be evaluation for female factor infertility with assessment of ovulatory status by history and laboratory testing (A). If these are normal or equivocal, the next step would be to proceed with assessing tubal patency through a hysterosalpingogram (C). A hysterosalpingogram will also give evidence of any anatomic uterine abnormality. If the first semen analysis is abnormal, repeat testing in days to weeks is warranted because of significant variability. If repeat testing reveals oligozoospermia once again, then investigation of male serum FSH, LH, and testosterone (B) is the next step. Telling the couple that they require a sperm donor (E) to achieve pregnancy is not appropriate without further testing and repeat semen analysis.
-
Question 17 of 100
17. Question
1 pointsA 43-year-old African American woman is concerned regarding breast cancer although she is normal. In past years, she had noted bilateral breast tenderness prior to her menses, but this has since abated. She had two cesarean deliveries, but no other surgeries. She takes a low-dose OCP. She does not smoke, and her family history is negative. All annual mammograms (since age 40) are negative. She wants to know whether BRCA1 and BRCA2 screening would be appropriate for her. Your reply is
Correct
Incorrect
Explanation:
Of the cases of breast cancer that are heritable, approximately 80% are due to mutations in BRCA1 and BRCA2. BRCA1 is associated with high risk for breast and ovarian cancer. BRCA2 is associated with a high risk of female and male breast cancer.
On the basis of our current understanding, however, less than 10% of all breast cancer cases can be considered to be heritable. Therefore, the total number of breast cancer cases associated with BRCA1 and BRCA2 mutations is a small percentage of the total number of breast cancer cases. Furthermore, there are numerous mutations that can occur in the BRCA1 and BRCA2 genes and can be related to an increased cancer risk. Some patients who have a mutation associated with cancer will not go on to develop cancer. Other patients may have a strong family history of breast cancer but no identifiable mutation. At present, therefore, screening of the general population is not recommended. This patient has no family history and is not in a high-risk group. Her prior breast tenderness was likely mastalgia related to the premenstrual phase. Therefore, BRCA1 and 2 screening would not be recommended for this patient. To state that BRCA1 and 2 screening should be performed after age 50 (choice B) is incorrect. As noted above, given the limitations of the testing for BRCA1 and 2 mutations, screening of the general population is not recommended. To state that BRCA1 and 2 screening should be performed if breast pain recurs (choice C) is incorrect. This patient does not need screening, not because her breast pain has resolved, but rather because BRCA1 and 2 screening is not appropriate for the general population at this time. As noted above, her breast pain was likely cyclic mastalgia secondary to hormonal changes prior to menses. To state that either BRCA1 screening (choice D) or BRCA2 screening (choice E) is recommended is not correct. As explained above, screening for neither of these is recommended.
-
Question 18 of 100
18. Question
1 pointsA 52-year-old woman has a receptor negative infiltrating carcinoma of breast with impalpable lymph nodes. Immunohistochemically, staining for HER2/neu (c-erb B2) is positive. Which of the following additional treatment options is most appropriate, based upon these findings?
Correct
Incorrect
Explanation:
This is an infiltrating carcinoma.
The lack of estrogen receptor staining suggests a poor response to hormonal therapy with tamoxifen.
The positive C-erb B2 (HER2/neu) staining suggest that trastuzumab (Herceptin) may be effective. -
Question 19 of 100
19. Question
1 pointsA 55 year old black female has undergone partial colectomy for cancer. The surgical specimen has clean margins, and there is no lymph node involvement. There is no evidence of metastasis. You recommend periodic colonoscopy for surveillance, and also plan to monitor which one of the following tumor markers for recurrence?
Correct
Incorrect
Explanation:
Prostrate-specific antigen (PSA) is a marker that is used to screen for prostate cancer. It is elevated in more than 70% of organ-confined prostrate cancers. Alpha-Fetoprotein is a marker for hepatocellucar carcinoma and nonseminomatous germ cell tumor, and is elevated in 80% of hepatocellular carcinomas. CA-125 is a marker for ovarian cancer. Although it is elevated in 85% of ovarian cancers, it is elevated in only 50% of early-stage ovarian cancers.
Carcinoembryonic antigen (CEA) is a marker for colon, esophageal, and hepatic cancers. It is expressed in normal mucosal cells and is overexpressed in adenocarcinoma, especially colon cancer. Though not specific for colon cancer, levels above 10ng/mL are rarely due to ~ benign disease. CEA levels typically return to normal within 4-6 weeks after successful surgical recurrence. Cancer antigen 27-29 (CA 27-29) is a tumor marker for breast cancer. It is elevated in about 33% of early-stage cancers and about 67% of late-stage breast cancers. Some tumor markers, such as CEA, Alpha-fetoprotein, and CA-125, may be more helpful in monitoring response to therapy than in detecting the primary tumor. -
Question 20 of 100
20. Question
1 pointsA 75 year old man undergoes transurethral resection of the prostate (TURP). Low grade carcinoma is seen in 5% of the specimen. What is the most appropriate intervention at this time?
Correct
Incorrect
Explanation:
In transurethral resection of the prostate (TURP), a thin cylindrical instrument (resectoscope) is inserted through the urethra where it is surrounded by prostate tissue. An electrical loop cuts away excess prostate tissue to improve urine flow.
In a man with advanced age if the tissue shows low grade (Stage T1a-Incidental histologic finding in 5% or less of the tumor resected). It is ok to observe without any further aggressive intervention. Options for more advance cases of prostate carcinoma include prostatectomy and radiation. -
Question 21 of 100
21. Question
1 pointsA 33 year old man presents to you with painless testicular swelling. Transillumination is done and is negative. The swelling subsides on lying flat. He remembers that he developed it after some minor trauma. What is the likely diagnosis?
Correct
Incorrect
Explanation:
hydrocele is fluid build-up around the testicle(s), while a varicocele occurs when blood backs up in the main vein that drains the scrotum. Varicocele is a condition in which the blood supply of the testis develops varicose veins. Veins contain valves that prevent blood from flowing backward. Faulty valves can result in a varicocele. Varicoceles usually develop on the left side of the scrotum and may produce no symptoms. Alternatively, varicoceles may cause pain and a sense of fullness that becomes bothersome. The varicocele feels like a bag of worms when the man is standing. However, the swelling usually disappears when he reclines because blood flow to the enlarged veins decreases. Rarely, a varicocele impairs fertility. During the transillumination test, a flashlight is shone through the enlarged portion of the scrotum. If a hydrocele is present, the scrotum will light up and the test will be positive. If it is a varicocele it will not light up. Germinoma occurs in females (dysgerminoma).
-
Question 22 of 100
22. Question
1 pointsA man aged 52 year has a large carcinoma of the distal esophagus. Associated weight loss of 9 kg is present. Preoperatively, he is started on full-strength enteral nutrition via a feeding tube placed in the stomach. Out of the following, which is the most common side effect of tube feedings?
Correct
Incorrect
Explanation:
Enteral nutrition is feeding a patient with specially prepared liquid feeds through a tube into your stomach (PEG tube) or small intestine (PEJ tube). Different formulas are available. Your dietitian will choose the most appropriate one for you, depending on your own particular nutritional needs. Some people may need to take all their food and drink this way. While others can take some food by mouth and some fed through the tube. Enteral feeding is only suitable for people whose digestive tract (stomach and intestines) is working normally. This is because the feeds still go through the usual path of digestion. Enteral feeds can cause side effects. Because they are quite concentrated, some of the feeds can cause sickness and diarrhea. If this happens, you may need to have your feeds more slowly. It is common to start off slowly and then build up from there. You may also need extra fluids through a drip to prevent dehydration from severe diarrhea.
-
Question 23 of 100
23. Question
1 pointsA man aged 52 year has a large carcinoma of the distal esophagus. Associated weight loss of 9 kg is present. Preoperatively, he is started on full-strength enteral nutrition via a feeding tube placed in the stomach. Out of the following, which is the most common side effect of tube feedings?
Correct
Incorrect
Explanation:
Enteral nutrition is feeding a patient with specially prepared liquid feeds through a tube into your stomach (PEG tube) or small intestine (PEJ tube). Different formulas are available. Your dietitian will choose the most appropriate one for you, depending on your own particular nutritional needs. Some people may need to take all their food and drink this way. While others can take some food by mouth and some fed through the tube. Enteral feeding is only suitable for people whose digestive tract (stomach and intestines) is working normally. This is because the feeds still go through the usual path of digestion. Enteral feeds can cause side effects. Because they are quite concentrated, some of the feeds can cause sickness and diarrhea. If this happens, you may need to have your feeds more slowly. It is common to start off slowly and then build up from there. You may also need extra fluids through a drip to prevent dehydration from severe diarrhea.
-
Question 24 of 100
24. Question
1 pointsA 26 year old male states that he experiences painful defecation in the anal region when he goes to toilet. You find a bluish purple mass at the anal margin on examination. What is the most likely diagnosis?
Correct
Incorrect
Explanation:
Hemorrhoids are dilated veins of the hemorrhoidal plexus in the lower rectum. Symptoms include irritation and bleeding. Thrombosed hemorrhoids are painful. Diagnosis is by inspection or anoscopy. Treatment is symptomatic or with endoscopic banding, injection sclerotherapy, or sometimes surgery.
External hemorrhoids may become thrombosed, resulting in a painful, purplish swelling. Rarely, they ulcerate and cause minor bleeding. Cleansing the anal region may be difficult.
Internal hemorrhoids typically present with bleeding following defecation; blood is noted on toilet tissue and sometimes in the toilet bowl. Internal hemorrhoids may be uncomfortable but are not as painful as thrombosed external hemorrhoids. Internal hemorrhoids sometimes cause mucus discharge and a sensation of incomplete evacuation. Strangulated hemorrhoids occur when protrusion and constriction occlude the blood supply. They cause pain that is occasionally followed by necrosis and ulceration. -
Question 25 of 100
25. Question
1 pointsPatients over the age of 65 are more likely to have which one of the following than younger patients, when presenting with appendicitis?
Correct
Incorrect
Explanation:
Older patients with appendicitis are more likely to present without classic signs and symptoms. Elevated WBC counts, rebound tenderness, guarding, and fever are less reliably seen. Perforation is found in over 65% of elderly patients at the time of diagnosis. The causes are delay in diagnosis and because vascular supply is already compromised due to atherosclerotic disease, if infective thrombosis occures, it will ace the perforation.
-
Question 26 of 100
26. Question
1 pointsA 22 year old man sustained a cut to his foot during beach vacations while wading. The cut wasn´t treated when it happened, and it is healing. He returned and presents to you saying that it feels like something in the wound is “poking” him. Which one of the following would be visible on a plain film radiograph?
Correct
Incorrect
Explanation:
Almost all glass is visible on radiographs if it is 2 mm or larger, and contrary to popular-belief, it doesn´t have to contain lead to be visible on plain films. Many common or highly reactive materials, such as wood, thorns, cactus spines, some fish bones, other organic matter, and most plastics, are not visible on plain films. Alternative techniques such as ultrasonography or CT scanning may be effective and necessary in those cases. Sea urchin spines, like many animal parts, have not been found to be easily detected by plain radiography.
-
Question 27 of 100
27. Question
1 pointsWhen repleting factor VIII in a hemophiliac in the midst of life-threatening bleeding, the minimum hemostatic level of factor VIII that is required is
Correct
Incorrect
Explanation:
A minimum of 80% of factor VIII is required for major surgery or life threatening bleeding. A minimum of 30% of factor VIII is required for mild hemorrhage, and 50% for treatment of joint and muscle bleeding. -
Question 28 of 100
28. Question
1 pointsA 33 year old female softball player notices a lump on her left breast. She reports to her doctor. Physical examination confirms the presence of a mass. The patient denies any family history of breast cancer and does not remember any recent trauma to the breast. Mammographic examination demonstrates some areas of focal calcifications. The biopsy of the mass demonstrates a prominent multinucleated giant cell reaction. The most likely diagnosis is which one of the following?
Correct
Incorrect
Explanation:
The correct answer is B. Multinucleated giant cells originate from histiocytes and participate in specialized forms of inflammatory reaction to certain pathogenic stimuli. Multinucleated giant cells are found for example, in granulomas developing in response to mycobacterial and fungal infections, lung injury due to inhalation of airborne inorganic dust, or unknown stimuli in sarcoidosis. In addition, a multinucleated giant cell reaction may result from focal necrosis of fat with release of lipids into the interstitium. The latter mechanism accounts for a marked chronic inflammatory response with formation of multinucleated giant cells in fat necrosis of breast. Frequently, this develops as a result of trauma (which the patient may have forgotten) or following surgery. A poorly circumscribed nodule that must be differentiated from a neoplasm develops in the breast. Biopsy shows this nodule to consist of a central core of lipid filled macrophages, including numerous multinucleated histiocytes, admixed with acute and chronic inflammatory cells and surrounded by a peripheral rim of fibrous tissue.
Acute mastitis is an infection characteristic of the lactating breast, which is made vulnerable to bacterial invasion by microscopic lesions of the nipple. Through this portal of entry, Staphylococcus aureus penetrates into the breast, inciting a marked acute purulent inflammatory response with abscess formation.
Fibroadenoma is the most common benign tumor of the breast, occurring usually before age 30. It is composed of a mixture or fibrous stroma and glandular tissue. The current view is that the true neoplastic population is the fibroblastic component. It appears as a well circumscribed mobile nodule within the breast. There is no inflammatory or giant cell reaction. Fibrocystic changes include a wide spectrum of morphologic alterations in which formation of small cysts; increases in the number of acini, and fibrosis are the main components. Unless associated with proliferative changes of epithelial cells in the terminal duct-lobular unit, fibrocystic changes are benign and do not predispose to cancer. No giant cell formation is seen. Sclerosing adenosis is part of proliferative disease of the breast. It is characterized by proliferation of acini (adenosis) and fibrosis and is associated with a slightly increased risk of developing cance -
Question 29 of 100
29. Question
1 pointsWhich of the following is the first step in performing an exploratory laparotomy for abdominal bleeding following blunt trauma once the abdomen is opened?
Correct
Incorrect
Explanation:
The first step is to palpate the aorta to ensure that adequate blood pressure is being maintained. Once it is established that there is adequate pressure, attention can be turned to looking for an injury to the major organs. Little information can be gained from palpating the vena cava.
-
Question 30 of 100
30. Question
1 pointsIn preventing recurrent bleeding in esophageal varices as a result of portal hypertension which of the following is effective
Correct
Incorrect
Explanation:
To prevent recurrent variceal bleeding, it is necessary to decompress the portal hypertension. Only shunt surgery effectively accomplishes this goal. Vasopressin, balloon tamponade, and sclerotherapy are effective treatments of acute bleeding, but recurrent bleeding is inevitable. Cimetidine is ineffective in preventing bleeding
-
Question 31 of 100
31. Question
1 pointsAccurate statement regarding treatment of pancreatic carcinoma is
Correct
Incorrect
Explanation:
Pancreatic carcinoma is a poorly treated and often deadly disease. Five-year survival is extremely poor. Ten to twenty percent of tumors are surgically resectable although this group does not demonstrate a significant increase in five-year survival. Palliative procedures are the mainstay of management. Stenting can be an effective adjuvant to relieve obstructive jaundice and cholangitis.
-
Question 32 of 100
32. Question
1 pointsA 45 Year old hypothyroid woman, a mother of 6 children presents to the OPD with RUQ discomfort. There is RUQ pain on palpation and simultaneous inspiration. This elaborates which of the following?
Correct
Incorrect
Explanation:
Pancreatitis can produce both a periumbilical hematoma (Cullen´s sign) and a flank ecchymoses (Grey-Turner´s sign). Homan´s sign refers to pain on dorsiflexion in the presence of a calf deep venous thrombosis, but is not very sensitive or specific. Abdominal pain under the right costal margin during simultaneous palpation and inspiration is Murphy´s sign, which can be found in acute cholecystitis. The psoas sign is when right leg extension produces abdominal pain, and is found in acute appendicitis (the appendix overlies the psoas muscle). Crohn´s disease, a subset of inflammatory bowel disease, is not commonly associated with any of these clinical signs.
-
Question 33 of 100
33. Question
1 pointsWhat is the treatment of choice for acute pulmonary embolism?
Correct
Incorrect
Explanation:
Heparin is the initial treatment of choice for an acute pulmonary embolism. Warfarin is used as the second arm of treatment. Streptokinase has not been shown to improve mortality in clinical trials. Aspirin and antibiotics are not indicated. -
Question 34 of 100
34. Question
1 pointsA 48 year old insulin dependent diabetic woman is unresponsive on 2nd post op day after surgery. Chest examination shows clear lungs and a regular heart rate and rhythm. Blood pressure is 80/40 mmHg. Arterial blood gas reveals no evidence of diabetic ketoacidosis. Which should NOT be part of the initial resuscitation?
Correct
Incorrect
Explanation:
In managing any unresponsive patient, maintenance of oxygenation and circulation is essential. This often requires oxygen support and IV fluids if the blood pressure is decreased. In post op patients, you must always be aware of the depressant effect of narcotics on respiratory rate, and treatment of an unresponsive patient with Narcan is appropriate. In the absence of diabetic ketoacidosis, IV administration of an ampule of 50% dextrose can be a life saving maneuver in the patient who is hypoglycemic. By the same token, the administration of insulin could kill this patient.
-
Question 35 of 100
35. Question
1 pointsFive patients visit the hospital with Five different cancers. Which will respond best to adjuvant chemotherapy? Correct
Incorrect
Explanation:
Breast cancer responds the best of all these tumors to the use of adjuvant chemotherapy after surgical resection.
-
Question 36 of 100
36. Question
1 pointsA male aged 56 years with a prosthetic heart valve needs to undergo urgent abdominal surgery to rule out an intra abdominal abscess. The patient is taking warfarin and his current prothrombin time is 21. The most appropriate course of action is
Correct
Incorrect
Explanation:
In the case of a patient in need of urgent surgery, fresh frozen plasma is the fastest and safest way to correct the prothrombin time. Non urgent surgery should be postponed, but urgent surgery, such as in this case where an abscess is suspected, should not be delayed. Protamine reverses heparin, although it has no effect on prothrombin time. Vitamin K will counteract the effect of coumadin, but it takes several days to work, and it makes anticoagulation after surgery extremely difficult. Packed red blood cell transfusion will not correct an abnormal prothrombin time.
-
Question 37 of 100
37. Question
1 pointsA resident of surgery aged 32 years was operating on an AIDS patient when he suddenly was stuck with a non sterile suture needle. Which one of the following is FALSE regarding seroconversion following occupational exposure?
Correct
Incorrect
Explanation:
The Public Health Service working group suggests that chemoprophylaxis be started promptly (within one to two hours) after exposure and that it be continued for four weeks, if possible. Health care workers with occupational HIV exposure should be given follow up medical evaluation and counseling. The risk of seroconversion after percutaneous injuries averages 0.3%; after mucous membrane exposure, it is about 0.1%, and after skin exposure, less than 0.1%. The risk of seroconversion increases if there is visible blood on the device causing the injury and the in- jury is deep. HIV antibody tests should be given at baseline and thereafter for the next six months. It may be possible to lower the risk further by administering zidovudine in combination with lamivudine or a protease inhibitor.
-
Question 38 of 100
38. Question
1 pointsA neonate, who was apparently normal at birth, develops persistent regurgitation and vomiting in the second and third weeks of life. Fever is not present and hematologic studies and blood chemistries are normal. Which therapy is most likely to be effective in his case?
Correct
Incorrect
Explanation:
The baby probably has congenital hypertrophic pyloric stenosis, which usually presents at several weeks of age. Partial surgical incision through the pylorus (pyloromyotomy) is usually curative. Antacids are beneficial in esophageal reflux and peptic ulcer disease. Barium enema can reverse intussusception in a child, but would not be therapeutic in this case. Gastric resection is not indicated since the much less invasive procedure of pyloromyotomy is actually more effective. Oral antibiotics are not indicated, since this is not an infectious process.
-
Question 39 of 100
39. Question
1 pointsA 34 year old woman presents to the emergency department about a “tumor in her pelvis.” Past medical history is significant for two cesarean sections and she had percutaneous uterine fibroid embolization from a right femoral artery approach approximately two week ago. On exam she has a reducible mass of the right groin that is inferior and lateral to the public tubercle and medial to the femoral vein. A healed groin puncture is noted on the right. The likely diagnosis is
Correct
Incorrect
Explanation:
The patient has a femoral hernia. This hernia forms by passage of a loop of bowel into the saphenous opening of the fascia lata and then through the cribriform fascia, to bulge anteriorly under the skin over the saphenous opening. They occur below the inguinal ligament. Femoral hernias are much more common in women than in men. The major risk of this type (and most other types) of hernia is strangulation of the herniated small intestine, which may result in death of the involved tissue with significant risk of subsequent life threatening peritonitis and sepsis. A direct inguinal hernia represents intestinal structures passing through a gap in the abdominal wall between transversus abdominis and the internal oblique muscles; these structures do not pass through the deep ring.
A femoral aneurysm is a well recognized complication of angiography and vascular access performed from this approach. During catheterization, the vessel wall is weakened and an outpouching, or aneurysm, results. These are potentially very dangerous. Not only can they rupture, but they can clot and send thrombus to the extremity, resulting in gangrene. Treatment involves surgery or percutanous induction of thrombosis using a variety of sclerosing agents, such as thrombin injection. An incisional hernia occurs where a surgical incision has weakened the abdominal wall. An indirect inguinal hernia represents extrusion of abdominal contents through the deep ring, through the inguinal canal to emerge at the superficial inguinal ring. During physical examination, the hernia can sometimes be reduced by the patient lying on his/her back; if the physician applies pressure to the site of the deep ring, reherniation will temporarily be prevented verifying the diagnosis of indirect inguinal hernia. -
Question 40 of 100
40. Question
1 pointsA 59 year old female is found to have a serum calcium level of 11.8 mg/dL. Repeated testing confirms values between 10.9 and 12.2 mg/L, and PTH level is elevated. She is asymptomatic, has no pertinent family history, and has no evidence of renal stones or bone disease. She declines elective parathyroidectomy and elects to have close medical follow-up. While doing so, it would advisable for her to be placed on which therapy?
Correct
Incorrect
Explanation:
The underlying pathology in this patient is most likely a single parathyroid adenoma, for which surgical removal is the only cure. However, estrogen-progestin therapy is beneficial in postmenopausal women with primary hyperparathyroidism because of its ability to reduce bone resorption and thus increase bone density and possibly lower serum calcium concentrations. Other drugs that inhibit bone resorption and thus can be used in primary hyperparathyroidism are bisphosphonates and raloxifene. Calcitonin (B) is useful in the acute treatment (after IV fluids and loop diuretics) of severe hypercalcemia or when the hypercalcemia is severely symptomatic. It effects are short lived and long-term therapy is not possible because resistance to its hypocalcemic effects develops within 1 or 2 days. Low calcium intake (C) is incorrect because it can result in increased hormone production and can accentuate bone disease. Calcium intake for this woman should be modest, but not low. Thiazides (D) are contraindicated because they can worsen hypercalcemia. When diuretics are used to control excessive or symptomatic hypercalcemia, furosemide is recommended, but never thiazides. Vitamin D analogs (B) are used in patients with hyperparathyroidism secondary to renal failure, in which case, the kidney can no longer add a hydroxyl group to vitamin D. This results in decreased calcium uptake from the GI tract In turn, PTH levels are elevated secondarily. Vitamin D analogs correct this problem. However, this patient has no problem with Vitamin D synthesis; giving her Vitamin D analogs would result in increased calcium absorption from the GI tract and would exacerbate the hypercalcemia.
-
Question 41 of 100
41. Question
1 pointsA 68 year old smoker and drinker male has rotten teeth, has a hard, fixed, 4 cm mass in his left neck. The mass is just medial to and in front of the sternomastoid muscle, at the level of the upper notch of the thyroid cartilage. It has been there for at least 6 months, and it is growing. Which one is the most appropriate next step in diagnosis?
Correct
Incorrect
Explanation:
In this setting, the clinical diagnosis is metastatic squamous cell carcinoma to a cervical lymph node, from a primary (or multiple primaries) somewhere in the mucosa of the aerodigestive tract. Endoscopy and biopsies should establish the diagnosis. Thyroid cancer could indeed metastasize to neck nodes, sometimes before the primary tumor is palpable. In this particular setting, however, a radionuclide scan (A) would be a very distant second choice. Sputum cytology and CT scan of the lungs (B) is another tempting thought for a smoker. But when lung cancer metastasizes to the cervical nodes, it affects the supraclavicular nodes, not the nodes higher up in the neck. Open biopsy of the neck mass is an absolute no-no, whether it is excisional (E) or incisional (D). Doing so will in fact confirm the diagnosis, but at the cost of contaminating the tissues and interfering with the placement of incisions for the definitive surgery. Furthermore, the location of the primary (or primaries) would not be established.
-
Question 42 of 100
42. Question
1 pointsA 60 year old male with liver disease has some anal discomfort and spotting on completing defection for 2 months that has gotten worse for past 2 days. Examination of the anal area shows dilated and bleeding veins that do not prolapse through anal canal. He is prescribed sitz bath and local anesthetics. The appropriate next step in management is
Correct
Incorrect
Explanation:
This question evaluates an important concept in medical treatment always try conservative managements first. In this case, the patient has already been prescribed local anesthetics and sitz baths, but he also needs high fiber diet since hemorrhoids are often caused by a history of constipation. Topical antibiotics are not necessary since the rectal and anal mucosa are highly vascularzed and have an extraordinary ability to heal (B). Hemorrhoids are classified as either internal or external, based on their location. They occur when the veins of either the internal or external hemorrhoid plexus become enlarged and the supporting muscles deteriorate. Both types are very common and are associated with increased hydrostatic pressure in the portal venous system, such as during pregnancy, straining at stool or cirrhosis (as in this case). The most common manifestation of internal hemorrhoids is painless, bright red rectal bleeding that occurs with bowel movements. Some complain of a feeling of incomplete evacuation. Definitive diagnosis is made by physical examination. If possible, the patient is asked to strain as if defecating and the anus is examined for relapsing veins. Anoscopy and proctoscopy may be performed to rule out coexisting pathology. The physician has an obligation to determine that there is nothing more serious going on with this patient. The patient did not offer these symptoms at his last physical, and waiting for 9 months (C) is unethical, since you have a diagnosis already. Because it will be necessary to rule out cancer, this patient will need a digital rectal examination, anoscopy, and flexible sigmoidoscopy. Once malignancy has been ruled out, conservative treatment with sitz baths, stool softeners, and stool-bulking agents may be used. Anal fissure (D) is not the correct diagnosis here because the patient does not present with the classic exquisite pain during defecation .Most of the time the bleeding presents as streaks on the stool surface. If anal fissure was the diagnosis, the analgesics might have easily controlled the pain. Thrombosis is one of the complications of hemorrhoids. This occurs when the hemorrhoid has prolapsed and the normal blood within the vein has become acutely thrombosed. This leads to severe pain. The treatment for an acutely thrombosed hemorrhoid is incision and extraction of the clot, followed by compression of the incised area (E). This patient´s pain does not seem this severe; so do not schedule surgery without trying medical treatment first.
-
Question 43 of 100
43. Question
1 pointsA 38 year old diabetic and hypertensive male presents with erectile dysfunction. It first manifested at a time of significant marital stress. His diabetes is well controlled. He was found to have testosterone levels of 557ng/dL (normal range: 437 to 707ng/dL) and nocturnal penile tumescence. The likely underlying cause of his current complaint is
Correct
Incorrect
Explanation:
Even though this patient has risk factors for organic impotence-atherosclerosis (A) and diabetes mellitus (B) the sudden onset and the continued presence of nocturnal erections point to a psychogenic problem. Gemfibrozil (C) is not associated with erectile dysfunction. Beta-blockers such as metoprolol (D) are associated with sexual dysfunction, but patients usually present with symptoms after having been treated for a long period of time. This patient has sudden onset dysfunction. Further, beta-blockers do not result in a loss of libido; they result in a loss of the ability to maintain/achieve erections.
-
Question 44 of 100
44. Question
1 pointsA 78 year old man has mid-abdominal colicky pain with nausea, anorexia, and vomiting for the past 2 hours. Past history is significant for mitral stenosis. His temperature is 98.6 °F, BP is 95/60 mm Hg, and pulse is irregular at 102/min. There is a mild diastolic murmur heard at the apex. He has distended abdomen, with diffuse tenderness and mild rebound. X-ray shows distended small bowel and distended right colon, up to the middle of the transverse colon. The appropriate next diagnostic step is
Correct
Incorrect
Explanation:
This patient´s past history of mitral stenosis and his irregular pulse points toward a diagnosis of atrial fibrillation (A Fib). A Fib is a common cause of mesenteric thromboembolism, as presented in this case. The best net step in the diagnosis of mesenteric thromboembolism is an arterial angiography. But since this is not an option, we should move forward with the best next step, which is an exploratory laparotomy. (A) A CT scan of the abdomen (or MRI, which was not offered as a choice) has excellent diagnostic yield when looking for mesenteric venous thrombosis. The clinical picture is most consistent with an arterial occlusion. A diagnosis of a venous occlusion would be more likely in a patient with a history of deep venous thrombosis or pulmonary embolism. Color Doppler ultrasound (B) also has excellent diagnostic capability if done by an experienced sonographer under ideal circumstances. But gas is the enemy of sonograms. Given the gas distention in this man, the circumstances are no longer ideal for the ultrasound. Laparoscopy (D) is less invasive than open laparotomy, but the necessary, insulation of the abdomen in an already distended individual, with marginal vital signs, is not advisable. There are clear signs of an acute abdomen that needs an emergent intervention, Upper gastrointestinal endoscopy (E) will not reach the affected area of the bowel (between the ligament of Treitz and the mid-transverse colon, the territory drained by the superior mesenteric vein). Lower gastrointestinal, endoscopy would be better, inasmuch as the right colon could be seen (and incidentally, rule out a cancer as the potential cause of the, cutoff of the colonic distention). In an emergency, the next step would be angiography, embolectomy or exploratory laparotomy as explained above. -
Question 45 of 100
45. Question
1 pointsA 59 year old alcoholic has severe abdominal pain, nausea and vomiting. His temperature is 101.8°F. He is treated in the ICU for 1 week, where he requires chest tubes for pleural effusions and is on a ventilator for several days. He improves sufficiently to be transferred to the ward. 3 days after leaving the ICU and about 2 weeks after the onset of the disease his temperature rise to 104.2°F. Labs show leukocytosis. The likely diagnosis is
Correct
Incorrect
Explanation:
The initial diagnosis was acute pancreatitis because of the symptoms of abdominal pain, nausea, vomiting and the finding of the Grey Turner sign (bruises on the flanks due to subcutaneous tracking of digested blood around the abdomen from the inflamed pancreas). A common complication of pancreatitis, and often the reason forth demise of patients, is the development of a pancreatic abscess. The timetable is usually about 10 to 14 days from the onset of the disease, and the initial manifestations are high-grade fever and leukocytosis (as observed in the laboratory studies). The next step in management would be CT scan of the abdomen to locate the abscess for drainage. Chronic pancreatitis (A) develops after several years of recurrent attacks of pancreatitis, and is characterized by steatorrhea, diabetes, and constant pain, which is not the case here. Pancreatic pseudo cyst (D) is another potential complication of pancreatitis. However, the manifestations are related to pressured symptoms from the fluid collection, there is no fever or leukocytosis, and the timetable for development is about 6 weeks from the onset of the disease. Pelvic abscess (D) and Subphrenic abscess (E) are indeed in the differential diagnosis, as they also present with fever and leukocytosis some 10 to 24 days from the original problem. But, the original problem for these patients is usually an infectious process in the abdomen, e.g., ruptured appendix or a perforated viscus. If the problem began with pancreatitis, and then there are signs of sepsis, the pancreas would be the logical place to harbor the pus.
-
Question 46 of 100
46. Question
1 pointsA 54 year old HIV positive man has a fungating mass growing out of the anus. It is not painful. For the past 6 months, he has noticed blood on the toilet paper, has lost weight and looks emaciated. On examination, the mass is easily visible. It measures 3.5 cm in diameter and is fixed to surrounding tissues. He also has rock hard, enlarged lymph nodes on both groins, some of them as large as 2cm in diameter. Which one is the most likely diagnosis?
Correct
Incorrect
Explanation:
The entire description is classic for anal cancer, but the clincher is the presence of metastasis in the inguinal nodes. Adenocarcinoma of the rectum (A) could look like this if it arose very low in the rectum, but it would not metastasize to inguinal nodes. Condyloma acuminatum (B) could give fungating masses, but it would not lead to cachexia and would not produce the rock hard inguinal nodes. Viral infections may precede the development of this kind of tumor (and could coexist with it), but it would be wrong to assume that all the patient has is the benign viral process. External hemorrhoids (C) are not fungating masses, and they are usually very painful. They do not lead to inguinal adenopathy or cachexia. Rectal prolapse (D) would produce a protruding mass with concentric mucosal folds, would come in and out with straining and would be a nuisance but it would not produce adenopathy and cachexia.
-
Question 47 of 100
47. Question
1 pointsA 20-year-old woman sustains severe head injuries in a road traffic accident.Investigations are normal except that sodium and chloride are elevated.Which one of the following statements is correct?
Correct
Incorrect
Explanation:
There is a marked hypernatraemia with elevated chloride but normal potassium and urea in a patient with severe head injuries. The likely cause of this presentation is diabetes insipidus. Urine osmolality is therefore likely to be low. You cannot say that she has a hyperchloraemic acidosis as you do not have her bicarbonate concentration but if you assume that the anion gap would be normal, that is 10-12, then this would suggest that the bicarbonate is elevated, suggesting either a metabolic alkalosis or respiratory acidosis with compensation. Although restoration of normal volume and osmolality is required, giving 5% dextrose may exacerbate any cerebral oedema and so correction should be gradual.
-
Question 48 of 100
48. Question
1 pointsA 76 year old man with chronic bronchitis develops a bulge in his left groin. On examination, an elongated swelling is seen above the medial end of the inguinal ligament. When the patient coughs, the swelling enlarges, but does not descend into the scrotum. Which one of the following is the most likely diagnosis?
Correct
Incorrect
Explanation:
An inguinal hernia usually produces a painless bulge in the groin or scrotum. If the hernia can slide back and forth, the bulge may enlarge when a man stands or strains. The bulge may get smaller or disappear when the man lies down. If the hernia is incarcerated, the bulge does not get smaller or disappear. Strangulated hernias usually become very painful within minutes or hours.
To diagnose an inguinal hernia, a doctor examines the groin and the area around it. For the examination, a man may have to stand and cough or strain. Coughing or straining produces pressure in the abdomen and makes a hernia more obvious. To determine whether the hernia is reducible or incarcerated, the doctor usually tries to push the loop of intestine back into the abdomen by pushing on the upper part of the scrotum with a gloved finger -
Question 49 of 100
49. Question
1 pointsAn infant suffers from second degree burns on the lower limb. Which of the following is the appropriate management?
Correct
Incorrect
Explanation:
First-degree burns are the most shallow (superficial). They affect only the top layer of skin (epidermis). Second-degree burns extend into the middle layer of skin (dermis). Third-degree burns involve all three layers of skin (epidermis, dermis, and fat layer), usually destroying the sweat glands, hair follicles, and nerve endings as well.
As with more superficial burns, deep minor burns are treated with antibiotic cream. However, any dead skin and broken blisters must be removed before the antibiotic cream is applied. In addition, keeping a deeply burned arm or leg elevated above the heart for the first few days reduces swelling and pain. The burn may require frequent re-examination at a hospital or doctor´s office, possibly as often as daily for the first few days. -
Question 50 of 100
50. Question
1 pointsWhich of the following determines most of the deaths?
Correct
Incorrect
Explanation:
The absence of vital signs is the commonest mode of diagnosing death and entails absent heart sounds, respiratory arrest and loss of papillary light reflex. Brain stem tests are generally reserved for ventilated cases where organ transplantation may be required. Doctors most commonly diagnose death not the coroner who provides a verdict on the cause of death.
-
Question 51 of 100
51. Question
1 pointsAn 8 year old girl has terminal bone cancer unresponsive to chemotherapy. She has been in the hospital for palliative care for just over 3 weeks. Today she is feeling tired, she is repeatedly complaining of pain in her leg, and wants to be held in her mother´s arms. She refuses to eat because according to her there is no point in eating if she will die anyway. The appropriate procedure for this patient is
Correct
Incorrect
Explanation:
Terminally ill patients often report that they fear intractable pain more than they do dying. Patients frequently express the desire to have open and honest dialogue about pain and want to be involved fully in planning their pain management. A doctor must be able to get alongside the patient and their family and to spend time presenting options, answering questions and quelling fears. Having promised to keep the patient relatively pain-free, it is paramount to meet this promise to preserve trust. Pain is easier to prevent than it is to relieve and drugs should be prescribed on a prophylactic basis with no other consideration than maintaining the patient´s quality of life. Pain is a complex subjective phenomenon and is affected by the emotional context in which it is endured. Adequate psychological support is critical as removing the fear of pain in itself will help to optimize pain control. A patient who is fearful, withdrawn and depressed often appears to have a lower pain threshold than one who is still actively engaged in enjoying what is left of their lives, even though they may have same stage disease. Non-drug measures to help psychological or spiritual distress may be as important as medication in relieving pain and suffering.
-
Question 52 of 100
52. Question
1 pointsWhich one of the following is correct regarding the routine use of prophylactic antibiotics in surgery?
Correct
Incorrect
Explanation:
Pre-operative antibiotics given prior to skin incision have significantly lowered surgical infection rates. Antibiotics should be given within two hours of surgery. There is no advantage to starting antibiotics 24 hours prior to surgery. Antibiotics can be continued 24 to 48 hours after surgery but should not be routinely continued for longer than this time period. Post-operative doses have not been shown to be as important as pre-op doses.
-
Question 53 of 100
53. Question
1 pointsA 58 year old man presents with an enlarged right inguinal lymph node on the medial side of the horizontal chain. Biopsy of the lesion demonstrates a well differentiated adenocarcinoma forming large glands and producing copious amounts of mucin. The most likely primary site for this cancer is
Correct
Incorrect
Explanation:
The lymph nodes of the groin are divided into superficial and deep groups. The superficial group is further divided into horizontal and vertical chains. It is worth remembering that tumors from the penis, vagina and anal canal can drain to the medial side of the horizontal chain of the superficial group of inguinal lymph nodes. Tumors from the ascending colon do not usually metastasize early to easily palpable lymph nodes; they instead metastasize to mesenteric nodes. Tumors from the duodenum do not usually metastasize early to easily palpable lymph nodes. They metastasize instead to the rich network of lymph nodes around the duodenum, pancreas, stomach, and liver. Tumors from the stomach can metastasize early to the easily palpable left supraclavicular nodes (also called Virchow´s nodes or sentinel nodes). They can also metastasize to nodes near the stomach. Tumors from the transverse colon do not usually metastasize early to easily palpable lymph nodes; they instead metastasize to mesenteric nodes.
-
Question 54 of 100
54. Question
1 pointsA female patient presents six months after uncomplicated caesarean section delivery of her first child with excruciating left lower quadrant abdominal pain and a palpable mass in that area that tents the skin. The pain is worse when she bears down with bowel movements and when she is on her feet all day. A hernia is palpated and easily reduced, which relieves her pain. What should be the next step in management?
Correct
Incorrect
Explanation:
Hernia following caesarean section is relatively common and may occur because of the increased elasticity and abdominal pressure that occurs during pregnancy. As long as the hernia remains easily reducible, it can be observed and electively repaired if symptoms dictate. There is no need to resect small bowel in a hernia without strangulation. Medication has no effect on the hernia. Emergent repair is only indicated in cases of incarceration and/or strangulation.
-
Question 55 of 100
55. Question
1 pointsA 52 year old man suffered a motor vehicle accident and is brought to the emergency department with severe abdominal pain. The abdomen is tender to touch. The patient´s blood pressure remains stable throughout the examination. CXR reveals no abnormality and abdominal films are inconclusive. Peritoneal tap yielded clear to straw colored fluid. Chemical analysis reveals the following:
Protein: low Bilirubin: low Amylase: low Lipase: low Red blood cells: low Ammonia: high Urea: high The most likely cause of the patient´s condition is
Correct
Incorrect
Explanation:
A direct blow to the bladder can lend to the extravasation of urine into the peritoneal cavity. This urine would have basically the same chemical characteristics of urine that is passed normally. Levels of urea, creatinine, and ammonia would be high. Amylase and lipase would below. In preparing for a peritoneal tap, it is important to ensure that, if possible, the bladder is empty. Accidental entry into the bladder would give similar results. Ascites from liver failure can have a straw colored appearance and be low in protein. The fluid would also be low in urea and creatinine. Depending on the serum ammonia levels, the ascitic fluid ammonia levels would range from low to elevated. Intestinal strangulation secondary to volvulus or hernia incarceration would lead to damage to the intestinal bowel wall. The fluid would be high in ammonia (and alkaline phosphatase) but low in creatinine and urea. Peritoneal effusions secondary to acute pancreatitis would have high levels of amylase and lipase. Perforated bile duct or for that matter, a perforated gallbladder, would give an effusion with a greenish hue. Urea and creatinine would not be elevated.
-
Question 56 of 100
56. Question
1 pointsIn preventing recurrent bleeding in esophageal varices as a result of portal hypertension which of the following is effective Correct
Incorrect
Explanation:
To prevent recurrent variceal bleeding, it is necessary to decompress the portal hypertension. Only shunt surgery effectively accomplishes this goal. Vasopressin, balloon tamponade, and sclerotherapy are effective treatments of acute bleeding, but recurrent bleeding is inevitable. Cimetidine is ineffective in preventing bleeding
-
Question 57 of 100
57. Question
1 pointsA patient develops a hernia on the lateral aspect of the rectus abdominus muscle. This is referred to as which one of the following?
Correct
Incorrect
Explanation:
A hernia is an abnormal protrusion of tissue through a fascial defect. A Spigelian hernia is an acquired hernia through the linea semilunaris (the lateral border of the rectus abdominus muscle). A Richter hernia occurs when part of the bowel becomes incarcerated in the defect. A direct hernia occurs because of a defect in the transversalis fascia in Hesselbach´s triangle, as opposed to an indirect inguinal hernia, in which the processus vaginalis fails to be obliterated. Littre´s hernia contains a Meckel´s diverticulum within it. A pantaloon hernia is a combined direct and indirect hernia.
-
Question 58 of 100
58. Question
1 pointsA 56 year old man is diagnosed with gastric carcinoma. Staging indicates that there is distant spread of the disease. What should be the appropriate management?
Correct
Incorrect
Explanation:
The survival rate for patients with distant spread of gastric carcinoma is poor. Patients with distant spread should receive palliation treatment only for such conditions as obstruction. Radiation and chemotherapy are not effective. Subtotal and total gastrectomy are indicated for localized tumors.
-
Question 59 of 100
59. Question
1 pointsA 33 year old man is in the ICU following a motor vehicle accident. He is in a coma and is receiving enteral feedings through a jejunostomy tube. Diarrhea starts to occur after several days. All of the following are associated with diarrhea secondary to enteral feedings, EXCEPT
Correct
Incorrect
Explanation:
Rapid bolus feedings in the supine position in a comatose patient can cause aspiration of the formula, and not diarrhea. Hypertonic feedings cause high osmolality and thus osmolar diarrhea. Bacterial contamination of feedings can cause infectious diarrhea. Refrigeration of formula and strict handwashing is necessary. Decrease in the acidity of the bowel by H2 blockers can alter the bowel flora and thus cause diarrhea. Antibiotics can also alter the bowel flora and cause C. difficile diarrhea.
-
Question 60 of 100
60. Question
1 pointsAn unrestrained front seat passenger in a car that crashes sustains closed comminuted fractures of both femoral shafts. He develops a BP of 80/50 mm Hg, a pulse of 110/min, and a venous pressure of zero. He becomes pale, cold, and clammy, but the rest of his examination and chest and pelvis X-rays are normal. Abdomen sonogram is also negative. The likely reason for the low BP is
Correct
Incorrect
Explanation:
Fractures of the femurs are known to be one of the few places in the body where enough occult blood loss may occur to lead to hypovolemic shock. Fat embolism (B) is also associated with long bone fractures, but the manifestations are those of respiratory failure, rather than hypovolemic shock. Neurogenic shock (C) would rarely occur from pain alone, being more common as a sequela of high spinal cord transaction. When it happens, the patient is hypotensive but looks warm and flushed rather than cold and pale. Intracranial bleeding (D) can lead to neurologic symptoms, but not to hypovolemic shock. There is not enough room within the head to accumulate the sizable blood loss required to go into shock. Pericardial temponade (E) would produce high central venous pressure.
-
Question 61 of 100
61. Question
1 pointsA 37 old man has right upper quadrant pain, nausea, and vomiting for 2 days. Exam reveals intense pain and inspiratory arrest on deep palpation of right upper quadrant. He is discharged after giving pain medication. Few days later he returns and complains of right upper quadrant pain. His temperature is 103.1oF. He has yellow coloration of the skin and sclera. His WBC count is 21,500/mm3, total bilirubin is 7.2 mg/dL, AST and ALT are normal. ALP is 512 U/L. LDH is elevated. The likely cause of his symptoms is
Correct
Incorrect
Explanation:
The patient has fever, right upper quadrant pain, and jaundice (together known as Charcot triad, characteristic of cholangitis), along with elevated alkaline phosphatase. Ultrasound would show dilatation of the common bile duct along with the extra and intrahepatic biliary apparatus, which would indicate that there is some obstruction at the level of the common bile duct. This obstruction is most likely due to a common bile duct stone (known as choledocholithiasis). Biliary obstruction is the primary etiology in most patients with acute cholangitis. All patients with suspected cholangitis should be evaluated for biliary obstruction by an imaging study. ERCP is both diagnostic and therapeutic. Acute hepatitis should be suspected in all patients with abdominal pain. However, the patient´s normal enzyme values and the high LDH and amylase values would not be explained by acute hepatitis (B). This patient probably presented initially with a biliary colic (A). However, at this time, the case is most consistent with a cholangitis due to the presence of jaundice and the high fever, which are unlikely to be seen in a cholelithiasis or a cholecystitis case. This patient probably had a gallbladder stone that traveled to the common bile duct, partially obstructing it, and leading to an ascending infection. Choledocholithiasis and gallstones both predispose patients to acute pancreatitis. However, the patient has a normal lipase level, and it is unlikely that he has acute pancreatitis (C). Amylase can be elevated with inflammatory intraabdominal pathology, including acute cholangitis. Peptic ulcer disease (PUD) (E) causes epigastric pain, but fever, jaundice, and biliary obstruction would not be explained by PUD.
-
Question 62 of 100
62. Question
1 pointsA 38 year old male wearing a cervical collar is brought to ED after being struck while crossing the street. He is agitated and confused, and smells of alcohol. Breath sounds are absent over the right hemithorax, so chest tube is placed. He is intubated. Findings on CT include fracture of the right transverse process of his T2 vertebrae and a small residual right hemothorax. Head CT shows a right frontotemporal contusion with a subarachnoid hemorrhage and multiple non-displaced calvarial fractures. Next day he becomes bradycardiac, his BP remains in the 130/80 mm Hg. Next step in his management is
Correct
Incorrect
Explanation:
This question describes a patient with intracranial hemorrhage and evidence of increased intracranial pressure. Cushing´s Triad refers to the bradycardia, hypertension, and irregular respiratory patterns with which this patient presents-obvious bradycardia and BP of 130/80 mm Hg is relative hypertension in a recent trauma patient. Furthermore, failure to over breathe the ventilator by a patient for whom sedation is not mentioned is considered irregular. This diagnosis of Cushing´s triad can be made clinically based on this combination of symptoms or can be detected on fundus examination, with a CT scan showing edema or midline shift, or with the insertion of an intracranial pressure monitor. Regardless of the method of diagnosis, the treatment is reduction of the increased intracranial pressure, which includes several strategies-from least invasive, including raising the head of the bed, sedation with propofol, hyperventilation to a PaCO2 of 30-35 mm Hg, and administration of mannitol (osmotic diuretic); to fairly invasive procedures including surgical intervention with a Burr hole decompression; or craniotomy and evacuation. In this case, the diagnosis can be made clinically, although a CT scan should be obtained, as ordered, to assess for progression of the known bleed. Treatment, therefore, is aimed at decreasing intracranial pressure, with the best option being the administration of mannitol. Atropine (A) is useful for bradycardia via parasympathetic antagonism; it is also useful in cardiogenic bradycardia, but not when due to an intracranial process, in which it will simply elevate the heart rate not treat the underlying problem. Emergent craniotomy (C) may become necessary, but is the most invasive strategy and should only be performed if less invasive techniques fail. There is no indication for removal of the cervical collar (D), which would in no way manage the underlying problem of elevated intracranial pressure. Mechanical ventilation is indicated for patients with increased intracranial pressure, as they can be sedated to decrease cerebral perfusion and thereby intracranial pressure. Accordingly, extubation (E) would be contraindicated.
-
Question 63 of 100
63. Question
1 pointsA 68 year old smoker and drinker male has rotten teeth, has a hard, fixed, 4 cm mass in his left neck. The mass is just medial to and in front of the sternomastoid muscle, at the level of the upper notch of the thyroid cartilage. It has been there for at least 6 months, and it is growing. Which one is the most appropriate next step in diagnosis?
Correct
Incorrect
Explanation:
In this setting, the clinical diagnosis is metastatic squamous cell carcinoma to a cervical lymph node, from a primary (or multiple primaries) somewhere in the mucosa of the aerodigestive tract. Endoscopy and biopsies should establish the diagnosis. Thyroid cancer could indeed metastasize to neck nodes, sometimes before the primary tumor is palpable. In this particular setting, however, a radionuclide scan (A) would be a very distant second choice. Sputum cytology and CT scan of the lungs (B) is another tempting thought for a smoker. But when lung cancer metastasizes to the cervical nodes, it affects the supraclavicular nodes, not the nodes higher up in the neck. Open biopsy of the neck mass is an absolute no-no, whether it is excisional (E) or incisional (D). Doing so will in fact confirm the diagnosis, but at the cost of contaminating the tissues and interfering with the placement of incisions for the definitive surgery. Furthermore, the location of the primary (or primaries) would not be established.
-
Question 64 of 100
64. Question
1 pointsA 55 year old man underwent a laparotomy 5 years ago for a gunshot wound. He now has vomiting and progressive abdominal distention for 5 days and since then has not passed stool or gas. He has hyperactive bowel sounds and abdominal discomfort, but no acute abdomen. AXR shows dilated loops, of small bowel, multiple air fluid levels, and no free air under the diaphragm. 6 hours later he develops fever, leukocytosis and abdominal tenderness. There is severe tenderness when external pressure is applied to his abdomen and then suddenly released. There are no audible bowel sounds. The appropriate next step in management is
Correct
Incorrect
Explanation:
He came in with mechanical intestinal obstruction due to adhesions, and has now developed signs of bowel strangulation. If the strangulated loop is still viable, it has to be freed immediately. If it is necrotic, it has to be respected with equal urgency to prevent continued peritoneal soiling. Antibiotics (A) will not provide viability to a compromised loop of bowel, nor prevent peritonitis if the loop is dead. Barium tag (B) is what we do in the postoperative period after abdominal surgery when we cannot decide whether a sluggish bowel has paralytic ileus or early mechanical obstruction. This is not the situation here. CT scan (C) is our universal answer when we do not know what is happening inside the belly. Here we do. Had he shown up with obstruction, and no reasonable etiology for it (no prior surgery, no hernias), we might have done CT. Endoscopy and a long tube (D) will not take care of dead or dying bowel.
-
Question 65 of 100
65. Question
1 pointsA 58 year old postmenopausal female noticed a lump in her left breast during self examination 2 days ago. She has no weight loss, bone pain, or anorexia. On exam her left breast has a palpable nodule in the upper outer quadrant. Mammography shows a speculated 8 mm density in same quadrant. Ultrasound guided biopsy shows an invasive ductal carcinoma that is estrogen receptor-positive, with a histology grade of 2 (out of 3). Which treatment regimen is appropriate at this time?
Correct
Incorrect
Explanation:
This patient has stage l breast cancer clinically, based on the size of the lesion and the lack of nodal involvement. Lumpectomy (breast conservation) has been shown to be as efficacious as radical mastectomy when combined with radiation therapy. Lumpectomy alone (A) is associated with higher rates of recurrence in patients with breast cancer. Lumpectomy alone can be performed in some women who fear cosmetic deformity but it is not recommended, due to the high rate of recurrence. Tamoxifen ought to be used if the tumor is estrogen receptor-positive, as it is in this case. Tamoxifen works as a competitive inhibitor of estrogen. It has been shown to reduce the recurrence and mortality when combined with radiation and lumpectomy (C) in this population of patients. Anastrozole (B) is also approved for the treatment of breast cancer in postmenopausal women; however, it has been shown to increase the risk of fractures. Since this patient has a history of osteoporosis, anastrozole is not preferred. Raloxifene, another selective estrogen receptor modulator (SERM) approved for the treatment of breast cancer, has been shown to decrease the risk of fractures in postmenopausal women and therefore can also serve as a preventive agent.
-
Question 66 of 100
66. Question
1 pointsA 38 year old diabetic and hypertensive male presents with erectile dysfunction. It first manifested at a time of significant marital stress. His diabetes is well controlled. He was found to have testosterone levels of 557ng/dL (normal range: 437 to 707ng/dL) and nocturnal penile tumescence. The likely underlying cause of his current complaint is
Correct
Incorrect
Explanation:
Even though this patient has risk factors for organic impotence-atherosclerosis (A) and diabetes mellitus (B) the sudden onset and the continued presence of nocturnal erections point to a psychogenic problem. Gemfibrozil (C) is not associated with erectile dysfunction. Beta-blockers such as metoprolol (D) are associated with sexual dysfunction, but patients usually present with symptoms after having been treated for a long period of time. This patient has sudden onset dysfunction. Further, beta-blockers do not result in a loss of libido; they result in a loss of the ability to maintain/achieve erections.
-
Question 67 of 100
67. Question
1 pointsA 24 year old man with neurofibromatosis type 1 presents with a left lower quadrant abdominal mass and signs of neurologic deficits in his left leg. He has higher than normal values of catabolites of epinephrine and nor epinephrine in 24 hour urinary collection. He is currently normotensive. Before invasive steps are taken, which one is the appropriate step in management?
Correct
Incorrect
Explanation:
The concern is that even though he is now normotensive, invasive steps might trigger a hypertensive crisis from the previously undiagnosed pheochromocytomas that he probably has. The presence of catabolites from epinephrine indicates that the tumor is in the adrenal glands, and not at an extra-adrenal site. Thus, the diagnosis of the pheochromocytomas can best be confirmed by MRI of the adrenals. Meningioma (A) and acoustic nerve tumors (C) occur in type 2 neurofibromatosis, not in type 1. Looking for pheochromocytomas outside of the adrenal glands (D) would have been a good idea if only elevated catabolites of nor epinephrine had been detected. The presence of high levels of epinephrine catabolites implicates the adrenal glands.
Radiation therapy (E) is a bad idea. Benign neurofibromas can be stimulated by radiation to undergo malignant transformation. -
Question 68 of 100
68. Question
1 pointsA 23 year old student has chest pain. It had sudden onset this morning after an episode of bilious, non bloody vomiting, he attributes to his eating “spoiled Chinese takeout” the night before. Medical history is significant for a small VSD managed since birth. He is tachypneic but has breath sounds bilaterally with a crunching noted over the left chest more than the right. His abdomen is slightly distended and his epigastrium region is tender to palpation. After sending lab tests the next management step is
Correct
Incorrect
Explanation:
The diagnosis is a perforated esophagus, also known as Boerhaave syndrome, based on the presence of vomiting, pain, and subcutaneous emphysema (Meckles triad). Forceful vomiting results in a transmural tear and perforation of the esophagus, usually in the left lateral position of the distal esophagus 3-5 cm above the gastroesophageal junction. Full-thickness transmural perforation is in contradistinction to Mallory Weiss syndrome, which is a tear of the inner layer of the esophagus, is not transmural, and usually presents as an upper GI bleed and not chest pain or subcutaneous emphysema. Hartmann sign is the presence of mediastinum crunching on auscultation and indicates the presence of pneumomediastinum. Approximately 90% of cases of esophageal perforation are iatrogenic during upper endoscopy, with the name “Boerhaave syndrome” reserved for spontaneous rupture typically due to vomiting. Diagnosis of an esophageal perforation is made on a Gastrografin swallow study, demonstrating contrast extravagating from the esophageal lumen. Gastrografin, which is water soluble, is not toxic to mediastinum and thoracic structures; as opposed to barium (A), which is more radiopaque and therefore more sensitive but also more dangerous. Treatment depends on the patient´s clinical situation and can include immediate primary repair via left thoracostomy if there is no gross contamination; drainage alone; conservative management with NPO status, intravenous fluids, and antibiotics; or diversion with a cervical esophagostomy.
Insertion of a nasogastric tube (C) would be of little immediate benefit and risks further damage to the perforated esophagus. It may be useful for gastric decompression and even feeding, if necessary, but plays no role in the acute setting. If feeding is desired and an operative intervention is pursued, a gastrostomy tube would be indicated. Transthoracic echocardiogram (D) would be of little help in diagnosing an esophageal perforation. The presence of a murmur and a known ventricular sepal defect is a distracter and is unlikely to account for the chest pain in this patient´s condition, given the other findings on physical exam. A transesophageal echocardiogram would obviously be contraindicated in the presence of a possible esophageal perforation. Upper endoscopy (E) is much more invasive than radiologic studies and carries its own risk of perforation. Additionally, direct visualization of the perforation contributes little to the ultimate management of the patient. -
Question 69 of 100
69. Question
1 pointsA 20 year old man is brought to the ER after sustaining a gun shot wound to his right lower back. He is alert and oriented. He is vitally stable, breathing without difficulty and has bilateral breath sounds. On exam he has a gunshot wound in the right lower back, just inferior to the sub costal margin 4 cm posterior to the posterior axillary line, oozing bright red blood. Abdomen is normal. Foley catheter yields clear urine. CXR reveals no pneumothorax, but the bullet is not visualized. The next step in the management is Correct
Incorrect
Explanation:
The patient is remarkably stable for a gunshot wound to the abdomen, so one must wonder if, in fact, he did sustain such injury. Only one bullet hole is noted, presumably the entry wound. There is, however, no exit wound. Given the musculature of the back, it is possible that the bullet did not traverse the superficial layers and enter the peritoneum or even the retro peritoneum. This would be consistent with his presentation. Although CT scan of the abdomen (B) or exploratory laparotomy (D) would prove this theory, the least invasive and quickest test is a cross table X-ray (C). It would likely reveal the bullet in a posterior position. If the bullet were in fact anterior, it would indicate that the bullet had traversed the abdomen and warrant either further imaging (such as a CT scan) or exploration, depending on the patient has reassessed clinical condition. This patient should not be assigned autocritically to exploratory laparotomy for a gunshot wound to the abdomen until it either is known, clinically or radiographically, that the gunshot was indeed to the abdomen. The findings of the chest X-ray are negative and there are no hemodynamic or respiratory findings that would suggest a thoracic injury. CT scan of the chest (C) is therefore incorrect. Insertion of Foley catheter shows clear urine, so an IV pyelogram (E) is not indicated.
-
Question 70 of 100
70. Question
1 pointsA patient has a prostatic specific antigen (PSA) of 6 g/L. A digital rectal examination (DRE) reveals a smooth, normal sized, non tender prostate. He is then sent for a transrectal ultrasound, which is reported to be normal. Which of the following would most significantly support performing a prostatic biopsy?
Correct
Incorrect
Explanation:
PSA is a glycoprotein produced only in the cytoplasm of prostate cells, and the serum level correlates with the volume of both benign and malignant tissue. As a first line screen, it is elevated in 10 to 15% of men self referred for screening. Prostatic cancer will be found in 18 to 30% of men with intermediate levels of elevation-4.1 to 10 g/L (normal is 4 g/L); 50 to 70% of those with levels 10 g/L will be found to have cancer. But 20% of patients who have radical prostatectomy for cancer will have a normal PSA. Treatment decisions in patients with untreated cancers cannot be made on the basis of PSA testing alone. Most cancers detected by digital rectal examination are advanced. Transrectal ultrasound should not be used as a first line screening tool because of its expense, and low specificity and, therefore, high biopsy rate. It increases the detection rate very little when compared with the combined digital rectal examination and PSA determination. PSA is not specific for cancer, and there is overlap of values in men with BPH and cancer. PSA detected cancers are more likely to be localized compared with those detected by rectal examination alone. The serial measurement of the PSA, called the PSA velocity, this may increase sensitively for cancer detection. Increased likelihood occurs with an increase of 0.75 g/L per year. A decrease in PSA velocity (A) is not usually associated with malignancy.
Further refinement may be done, however, in a patient with a normal rectal exam, and elevated PSA, and a normal transrectal ultrasound by calculating the PSA density. This is the serum PSA volume of the prostate as measured by ultrasound. Patients with high PSA densities are more likely to have cancer in spite of a normal rectal examination and ultrasound. The most recent refinement of PSA has been the measurement of free serum and protein-bound levels. Cancer patients have lower percentages of free serum PSA (C) as compared to protein bound PSA which is usually higher (D). Optimal cutoff levels have not yet been determined. -
Question 71 of 100
71. Question
1 pointsA 28 year old man underwent exploratory laparotomy for a stab wound. After 5 days large amounts of clear, pink fluid are soaking his wound dressings. On exam the incision appears intact and not particularly red or inflamed, but there are traces of the clear pink fluid on his skin. He is NPO and on IV fluids, but has been passing gas per rectum, and plans had been made to feed him today. Abdomen is not distended. The appropriate initial step in management is
Correct
Incorrect
Explanation:
The situation described is that of a wound dehiscence that has not yet progressed to a wound evisceration. The former can be dealt with conservatively, like any other wound, by taping the wound securely maintaining moist environment pain reduction and promotion of granulation tissue. The patient will eventually require re-closure, but this can be done whenever it is most convenient Cultures and antibiotics (A) would be appropriate it there was a sign of infection. Assuming the pink fluid to be sign of infection is not correct. It is normal peritoneal fluid (with a trace of blood still in it from the recent surgery) that is seeping out through the unhealed wound. Probing (B) will not produce pus, but it might cause evisceration. The patient is afebrile and the wound is not red. Clear pink salmon-colored fluid means dehiscence, not infection. Betting the patient out or bed (C) is absolutely inadvisable. If this were done, the person helping the patient would soon be looking at a portions or small bowel extruding from the abdomen. Taking the patient to the OR immediately (D) would be appropriate if the patient had wound dehiscence with evisceration.
-
Question 72 of 100
72. Question
1 pointsAn 80 year old man undergoes surgery for a fractured femoral neck. On 5th post-op day, it is noted that his abdomen is grossly distended and tense, but not tender. The rectal vault is empty on digital examination. X-ray shows few distended loops of small bowel and a much distended colon. The cecum measures 9 cm, and the gas pattern of distention extends throughout the entire large bowel, including the sigmoid and rectum. Other than the abdominal distention he does not appear to be ill. The most likely diagnosis is
Correct
Incorrect
Explanation:
Ogilvie syndrome is a type of colonic dysfunction often seen in elderly patients who are not too active to begin with and are then further immobilized by extra-abdominal surgery. Colonoscopy should be done to rule out obstructing cancer (always a consideration in this age group) and allows the gas to be sucked out as the scope is advanced. A long tube is then left in place. Fecal impaction (A) is always a good thing to look for in old immobilized people with abdominal distention. However, the rectal vault would have been full, and the X-ray films might have shown the fecal column extending up into the sigmoid and descending colon. Neither mechanical intestinal obstruction (B) nor paralytic ileus (D) would develop from hip surgery. When these complications occur after abdominal surgery they affect primarily the small bowel, not the colon. Volvulus of the sigmoid (E) is another good thought in the distended old patient but the radiologic picture would have been different with a huge distended sigmoid way up into the right upper quadrant and tapering toward the left lower quadrant with the classic image of a parrot´s beak.
-
Question 73 of 100
73. Question
1 pointsAn 18 year boy is brought after sustaining a fall. He landed on his right leg and heard a snap. He felt severe pain in his thigh and noticed that it appeared deformed. On exam there is extreme tenderness to movement and palpation of the right thigh and marked swelling over the region. The compartmental pressure of the right thigh is 15 mm Hg. His popliteal and pedal pulses on the right are barely palpable. X-ray shows a comminuted fracture through the right femoral diathesis with lateral angulations. After base line studies the next step in management is
Correct
Incorrect
Explanation:
This patient has injured his femoral artery with a bone fragment from the fractured femur. Vascular injury can be a sequel of femoral fractures, posterior knee dislocation, or fracture of the proximal tibia and fibula. A thorough lower extremity pulse examination is required in case of any of these injuries. Angiography may reveal an area of active bleeding or vascular disruption that may be embolized or identified as site for a possible surgical bypass. An emergent fasciotomy (B) should be used in acute compartment syndrome; the compartment pressure should be above 30 to 40 mm Hg. Reduction of the fracture (C) can be performed after vascular integrity has been assessed. Right leg amputation (D) is unnecessary at this time, as the leg may be salvaged with an intervention after angiograph. Vigorous hydration (E) would be necessary in the case of hypovolemia.
-
Question 74 of 100
74. Question
1 pointsWhich of the following does not have a role in the management of chronic cancer pain?
Correct
Incorrect
Explanation:
Pinavarium is used to reduce the pain duration in irritable bowel syndrome (IBS). Carbamazepine is in use for the treatment of neuropathic pain of malignancy, diabetes and other disorders. Clodronate inhibits osteoclastic bone resorption and is used to treat malignant bone pain and the associated hypercalcaemia. The corticosteroids are used to treat pain from central nervous system tumours.
Reducing the inflammation and oedema relieves the pain caused by neural compression. Nifedipine helps relieve painful oesophageal spasm and tenesmus associated with gastrointestinal tumours. Painful bladder spasm may be relieved by oxybutinin. -
Question 75 of 100
75. Question
1 pointsThree days post-operation of a sigmoid colectomy, a 65-year-old diabetic complains of dizziness and faintness.Blood pressure is 81/53 mmHg, pulse is 113 bpm, respiratory rate of 24/min, and SpO2 of 99% on air. His blood glucose is 18mmol/L (3.0-6.0 fasting). Electrocardiogram shows ST depression of 2 mm in leads II, III and AVF.Which of the following is the initial drug therapy for this patient?
Correct
Incorrect
Explanation:
Initial treatment of an acute coronary syndrome is aspirin 300 mg. This should be safe in the postsurgical patient with no signs of bleeding at 3 days post-operation. Clexane would also be given, but aspirin initially.
The dose of clopidogrel is 300 mg in an acute coronary syndrome. Diamorphine is used to treat anxiety and pain, neither of which is commented upon.
GTN would be reasonable to try, however the blood pressure is low. Remember that in the diabetic, chest pain may not be a feature of acute coronary syndrome due to autontmic dysfunction, and in most postsurgical patients myocardial infarct is silent. -
Question 76 of 100
76. Question
1 pointsA 16 year old black male presents to the emergency department with a chief complaint of fever, abdominal pain, nausea, and anorexia. In addition to the usual laboratory evaluation, which one of the following imaging study will confirm your diagnosis of appendicitis?
Correct
Incorrect
Explanation:
A retrospective review of 650 patients with suspected appendicitis showed a sensitivity of 97% and a specificity of 98% for spiral CT. In patients in whom the clinical diagnosis was uncertain, sensitivity was 92% and specificity was 85%. Two prospective studies comparing ultrasonography with spiral CT have favored spiral CT. Ultrasonography is used in women who are pregnant and women in whom there is a high degree of suspicion of gynecologic disease. Abdominal radiography has low specificity and sensitivity for the diagnosis of acute appendicitis. Air contrast barium enema also has low accuracy. Limitations of MRI include increased cost, decreased availability, and increased examination time compared to CT
-
Question 77 of 100
77. Question
1 pointsA 57 year old male, known case of cirrhosis comes to the hospital because he has been vomiting blood. Which diagnostic procedure is most useful in identifying the bleeding source? Correct
Incorrect
Explanation:
Varices are dilated veins in the distal esophagus or proximal stomach caused by elevated pressure in the portal venous system, typically from cirrhosis. They may bleed massively but cause no other symptoms. Diagnosis is by upper endoscopy (Esophagogastroduodenoscopy or EGD). Treatment is primarily with endoscopic banding and IV octreotide. Sometimes a transjugular intrahepatic portosystemic shunting procedure is needed.
-
Question 78 of 100
78. Question
1 pointsA 39 year old man is found lying on the floor unconscious from a drug overdose and is admitted to the hospital. The next morning, he is alert and complains of constant pain in the anterior aspect of his left leg. On examination, you find pain with passive stretching, pain with palpitation anteriorly, and a slightly decreased dorsalis pedis pulse on the left. No edema or erythema is present. X-rays of the lower extremity are normal. How would you proceed now?
Correct
Incorrect
Explanation:
The patient has a history and symptoms very suggestive of a compartment syndrome of the anterior compartment of the left leg. A common cause of this serious problem is limb compression during prolonged recumbency related to drug or alcohol overdose. Early diagnosis and treatment are essential in order to avoid permanent, severe disability.
The five “P´s” of compartment syndrome are pain, pallor, paresthesias, pulselessness, and paralysis. If distal pulses are reduced, then muscle necrosis has occurred and immediate surgical consultation is necessary. Duplex Doppler ultrasonography and impedance plethysmography are used to evaluate for deep venous thrombosis. Given the decreased pulses, however, this diagnosis is much less likely and such testing may needlessly delay urgent surgical treatment. Intravenous antibiotics would be appropriate for cellulites. This diagnosis is unlikely in this scenario given the absence of fever, erythema, and edema, and the presence of a diminished pulse. Rest, ice, and elevation are inappropriate treatment modalities for compartment syndrome. -
Question 79 of 100
79. Question
1 pointsThe treatment of choice for a rectal carcinoma that is 3 cm from the anal verge without evidence of metastatic disease is
Correct
Incorrect
Explanation:
Tumors within 5 cm of the anal verge should be treated with abdominal perineal resection with permanent colostomy. Hemicolectomy is indicated for right and left colon tumors. Anterior resection with anastomosis to the rectal stump is best reserved for sigmoid and upper rectal tumors. Total colectomy alone would not treat distal rectal disease. Palliation therapy is indicated for metastatic disease.
-
Question 80 of 100
80. Question
1 pointsThe treatment of suppurative cholangitis consists of
Correct
Incorrect
Explanation:
Suppurative cholangitis can result in life-threatening sepsis and requires IV fluids and antibiotics along with emergent drainage of the biliary tree. Observation and oral antibiotics are not effective and any delay in treatment can increase mortality which is as high as 50% in this disease.
-
Question 81 of 100
81. Question
1 pointsIn advising a patient diagnosed with breast carcinoma with positive nodes, which one of the following statements is correct?
Correct
Incorrect
Explanation:
While recent technological advances have led to earlier detection of breast cancer, this has not led to an absolute cure and a marked increase in survival. The major advance has been in demonstrating that lumpectomy with axillary node dissection offers about the same five-year survival as mastectomy The choice between these treatments should be determined on an individual treatment basis, with the psychological implications of mastectomy weighed against the fear of recurrence if the breast is not removed.
-
Question 82 of 100
82. Question
1 pointsAn 18 year old boy develops a painless, firm mass beneath the nipple of his left breast. The mass is mobile, and no fluid can be expresses from the breast. The right breast is normal to examination. This mass most likely represents which one of the following?
Correct
Incorrect
Explanation:
The most common breast mass in men, especially under 25 years of age, is gynecomastia, which is a benign proliferation of ductal and stromal elements of the breast. It is generally an idiopathic condition, probably related to pubertal hormonal changes. Gynecomastia can involve either one or both breasts and, if it arises during adolescence1wilI often resolve spontaneously with time. Fibrocystic changes in the breast, which reflect physiological responses in the breast to cyclical levels of sex hormones, are not observed in men. Fibrocystic changes are most common in the late reproductive years and include fibrosis cyst formation, and a variety of epithelial changes such as hyperplasia and apocrine metaplasia. Intraductal papillomas are benign neoplasms commonly evolving in the major lactiferous ducts beneath the nipple. They most commonly present with a bloody nipple discharge and are rare in men. Carcinoma of the male breast is rare and almost always develops in the breasts of elderly men. Male breast carcinomas have a somewhat worse prognosis than their female counterpart. Grossly and microscopically they usually resemble ductal carcinoma in the female. The male breast does not develop lobules and so carcinomas resembling lobular carcinoma are extremely rare in the male breast.
-
Question 83 of 100
83. Question
1 pointsFour days after abdominal surgery a patient´s hemoglobin dropped from 10 g/cm2 to 8 g/cm2. She is transfused two units of packed red blood cells and her post-transfusion hemoglobin is still 8 g/cm2. She is hemodynamically stable. Management at this time should consist of
Correct
Incorrect
Explanation:
The patient most likely has a gastrointestinal bleed. While she may be bleeding from a site other than the Gl tract, the Gl tract should be the first place checked. Her unchanged hemoglobin despite two units of blood is suggestive of an active bleed. Observation and IV fluids do nothing to address the source of bleeding. Fresh frozen plasma is not indicated, as there is no evidence of a coagulation abnormality. As the patient is stable, there is no indication for surgical exploration.
-
Question 84 of 100
84. Question
1 pointsA patient is brought to the emergency department after an automobile accident. There is ecchymosis of the right side of the abdomen and the patient is complaining of abdominal pain. What is the appropriate management? Correct
Incorrect
Explanation:
In cases of blunt abdominal trauma, diagnostic peritoneal lavage is the correct intervention. Exploratory laparotomy is indicated if there are >l00,000 red blood cells. If lavage is negative, observation for 24 hours is indicated.
-
Question 85 of 100
85. Question
1 pointsDrug pair that is most likely to be used for the treatment of testicular cancer is
Correct
Incorrect
Explanation:
Etoposide and cisplatin typically represent the combination chemotherapy that is used for the treatment of testicular cancer. Fluorouracil with the adjuvant levamisole is used for colorectal carcinoma. Methotrexate and dactinomycin are used in the treatment of choriocarcinoma, while vincristine and dactinomycin are used in patients with Wilms tumor.
-
Question 86 of 100
86. Question
1 pointsWhat is the most common clinical manifestation of pulmonary embolism?
Correct
Incorrect
Explanation:
The most common clinical sign of pulmonary embolism is tachypnea. Other important features include chest pain, dyspnea, an accentuated S2 heart sound, pulmonary rales, apprehension, tachycardia, fever, phlebitis, hypotension/syncope, and cyanosis.
-
Question 87 of 100
87. Question
1 pointsA male patient presents with a soft tissue sarcoma of an extremity. What is the appropriate treatment?
Correct
Incorrect
Explanation:
Wide resection is the treatment of choice for established soft tissue sarcomas. Wide resection consists of excision with a margin of normal tissue. Incision and excision are inadequate for malignant tumors. Marginal excision consists of excision through the reactive zone of tissue. This is not recommended for malignant tumors, although it is adequate for benign, but locally aggressive, tumors. Radical resection consists of resecting the entire compartment. This is overly aggressive and has not been shown to improve the outcome.
-
Question 88 of 100
88. Question
1 pointsA 19 year old girl is diagnosed with Hodgkin´s disease. In the future she wishes to have children. Radiotherapy is the recommended treatment. What should be the appropriate management at this time?
Correct
Incorrect
Explanation:
Laparotomy allows accurate staging and allows for transposition of the ovaries out of the radiation field, which is desirable if the patient wishes children in the future. Treatment should not be delayed, as this could lead to further spread of the disease. Chemotherapy will prevent a future healthy pregnancy. Progesterone will have no protective effect.
-
Question 89 of 100
89. Question
1 pointsWhich of the following is NOT an indication for surgery on a herniated disc in the lumbar spine?
Correct
Incorrect
Explanation:
Indications for lumbar disc decompression include progressive weakness and loss of reflexes, which are indicative of severe nerve compression. An inability to control urination is indicative of cauda equina syndrome, which is a surgical emergency. The real surgical dilemma occurs in the management of patients with pain as the chief complaint. Isolated low back pain does not usually respond to lumbar disc decompression, as the pain is most often due to mechanical back pain. These patients should be managed conservatively for as long as possible with a minimum of three months of conservative management. Patients with leg pain worse than back pain may be appropriate candidates for decompression after an initial period of at least six weeks of conservative care.
-
Question 90 of 100
90. Question
1 pointsn indication for peripheral vascular bypass surgery is which one of the following?
Correct
Incorrect
Explanation:
A nonhealing ulcer is an indication for peripheral revascularization, as the ulcer will eventually become infected and can lead to partial amputation without an adequate blood supply. Vascular claudication alone is not an indication for surgery. Neurogenic claudication is a result of spinal stenosis and will not be treated by vascular surgery. Weakness of dorsiflexion is not a common symptom of peripheral vascular disease and loss of sensation in the foot should make you suspicious of a diabetic neuropathy which often occurs in association with peripheral vascular disease.
-
Question 91 of 100
91. Question
1 pointsA month old child develops vomiting of increasing severity after feeding, eventually developing forceful vomiting. The vomitus contains milk and mucus, but not bile. Child also fails to gain weight and becomes constipated. Physical examination performed after feeding demonstrates visible waves of peristalsis traveling from left to right in the epigastrium. Which of the following techniques would be most useful in correcting in the baby´s problem?
Correct
Incorrect
Explanation:
The history is classic for “congenital” pyloric stenosis, which typically presents from 3 to 6 weeks of life. A small mass can be felt deep to the right of the rectus abdomen is when the stomach is empty which is the hypertrophied pylorus. This problem can be easily surgically corrected with pyloromyotomy, in which the pyloric muscle is partially cut, relieving the obstruction. Barium enema is occasionally effective in reducing childhood intussusception, but would not be of value in pyloric stenosis. Duodenoduodenostomy is used to correct congenital duodenal obstruction, but is not required for congenital pyloric stenosis. Gastric resection would be completely unnecessary in this case. Volvulus usually involves the small intestine and can produce an “acute abdomen” secondary to infarction of the bowel.
Milk is vomited and it maybe curdled or not. Depends on the time difference between feed and vomitus. If enough time exists for milk to mix with acid, it will be curdled milk vomiting. Or otherwise.
-
Question 92 of 100
92. Question
1 pointsegarding bariatric surgery the correct statement is which one of the following?
Correct
Incorrect
Explanation:
Bariatric surgery is a major gastrointestinal procedure. Bariatric surgery in adolescents raises social, psychological and developmental issues, but adolescents are not excluded from surgery, and some hospitals have specialised programmes for younger patients. Potential candidates for surgery are those with a body mass index (BMI) exceeding 40, or BMI > 35 with serious co-morbidities (eg sleep apnoea, type 2 diabetes). Post-operative mortality ranges from 0.1-2 %. Vomiting is a risk associated with bariatric surgery, as is dumping syndrome and nutritional deficiencies. There is no evidence as yet; that bariatric surgery reduces cardiovascular mortality in patients.
-
Question 93 of 100
93. Question
1 pointsA 45 year old female is recovering from a mild episode of acute ascending cholangitis secondary to choledocholithiasis. While awaiting cholecystectomy, she developed fever and leukocytosis and LFTs showed bilirubin of 2.5 mg/dL and alkaline phosphatase of 115 U/L. Sonogram shows a 6 cm abscess in the right lobe of the liver. She denies any recent travel. The appropriate treatment is
Correct
Incorrect
Explanation:
Liver abscess complicating biliary tract disease is described as pyogenic abscess (to contrast it with amebic abscess), and it requires drainage. The percutaneous route is favored. The difference between amebic and pyogenic abscess may be radiographically indistinguishable. Clinical suspicion based on age and epidemiologic risk factors is therefore most important. Those who have pyogenic liver abscesses are most likely to have a greater degree of jaundice, bacteremia/sepsis, and history of gallbladder disease or surgery. Amebic abscess should be suspected in young male patients who have recently traveled to endemic areas or are immigrants of endemic areas. Amebic abscess, when suspected, does not require immediate percutaneous drainage; it should be treated empirically with metronidazole (C) and have serologic testing performed (for E. histolytica). If the patient does not improve over a course of days, percutaneous drainage may be required. Endoscopic retrograde cholangiopancreatogram (ERCP) (A) is often urgently needed to treat acute ascending cholangitis, but it does nothing for a liver abscess. Long term antibiotics (B) do not penetrate an existing abscess. Abscesses have to be drained for resolution to occur, except in the case of amebic abscess, as described. Open surgical drainage (D) is not preferred for a liver abscess. If this option had been open surgical drainage plus removal of the gall bladder, this would have been a good option. Open surgical drainage alone, however, is more invasive than percutaneous intervention.
-
Question 94 of 100
94. Question
1 pointsesult is 8ng/mL. A digital rectal examination shows an area of induration within the prostate. The patient has been asymptomatic. If this patient´s confirmatory diagnostic test shows positive for localized disease, which one would be the most appropriate treatment?
Correct
Incorrect
Explanation:
Prostate cancer is the most common cancer detected in American men. Most of these cancers are small and contained within the prostate, and few are associated with regional or distant disease. Incidence increases with age. Obstructive voiding symptoms are most often due to benign prostatic hypertrophy (BPH), but large, locally extensive cancer can cause obstructive voiding symptoms. On digital rectal exam, prostate cancer may show as focal nodules or areas of induration; however, many are associated with normal palpation and so are detected on the basis of elevation of the serum PSA. Transrectal ultrasound guided biopsy is the best method for detection of prostatic cancer and allows for accurate grading. The biopsy is done in men with an abnormal digital examination or PSA. Early adenocarcinoma of the prostate represents disease that is localized to the gland without extension beyond the capsule or to distant sites. Following the establishment of the diagnosis, the patient should be prepared for a radical prostatectomy. Disease that includes or extends beyond the capsule is considered advanced carcinoma, and other treatment modalities should be sought (A, B, or C). Transurethral resection of the prostate (TURP) (E), is the most common surgical procedure to treat BPH. It has no use in the treatment of adenocarcinoma of the prostate.
-
Question 95 of 100
95. Question
1 pointsA 25 year old man has swelling in the scrotum and pain that travels toward left flank. He has a temperature of 100.8oF. There is distinct swelling of the left side of the scrotum proximally. Both testes are normal on palpation but there is a tender mass just above the left testes. He also has mild left flank tenderness. Prostate is mildly tender on rectal examination. A small cloudy urethral discharge is present. The most appropriate treatment is
Correct
Incorrect
Explanation:
There are two types of epididymitis: sexually transmitted and non-sexually transmitted. Both are infectious and the organism gains entry via the urethra. The sexually transmitted organisms are Chlamydia trachomatis and Neisseria gonorrhea. This form is more common in younger, sexually active men. The non-sexually transmitted type is seen more commonly in older men and the most common organisms are gram negative rods. This presents as a urinary tract infection with prostatitis. There is generally a fever present in the sexually transmitted form, the urethra will be involved and there may be a discharge, as in this patient. The main characteristic will be scrotal pain, usually of less than 12 hours duration, which may be localized to the top of the testis. The pain travels up the spermatic cord toward the flank, which causes abdominal pain. Patients may also present with nausea and vomiting. At first the inflamed epididymis may be palpated separately from the testis, but with time, there is just one large, swollen mass. Prehn sign consists of pain relieved upon elevation of the scrotal contents in patients with epididymitis. If there is any question regarding the diagnosis, a scrotal ultrasound may be helpful. The patient must be treated with antibiotics for 14-21 days and testicular elevation. Other measures, such as analgesics, the use of ice packs locally, and bed rest are also important. If sexually transmitted disease is suspected as the cause, you should also treat the sexual partner. Options A and D are important measures in the treatment of acute epididymitis; however, analgesics and ice packs must be administered in conjunction with antibiotics and scrotal support. Never attempt to percutaneously drain a mass in the testicles (C) without a confirmatory ultrasound. This is to avoid a lesion of the hemato testicular barrier or the spread of any possible malignancy in the testicle. -
Question 96 of 100
96. Question
1 pointsA 59 year old female is found to have a serum calcium level of 11.8 mg/dL. Repeated testing confirms values between 10.9 and 12.2 mg/L, and PTH level is elevated. She is asymptomatic, has no pertinent family history, and has no evidence of renal stones or bone disease. She declines elective parathyroidectomy and elects to have close medical follow-up. While doing so, it would advisable for her to be placed on which therapy?
Correct
Incorrect
Explanation:
The underlying pathology in this patient is most likely a single parathyroid adenoma, for which surgical removal is the only cure. However, estrogen-progestin therapy is beneficial in postmenopausal women with primary hyperparathyroidism because of its ability to reduce bone resorption and thus increase bone density and possibly lower serum calcium concentrations. Other drugs that inhibit bone resorption and thus can be used in primary hyperparathyroidism are bisphosphonates and raloxifene. Calcitonin (B) is useful in the acute treatment (after IV fluids and loop diuretics) of severe hypercalcemia or when the hypercalcemia is severely symptomatic. It effects are short lived and long-term therapy is not possible because resistance to its hypocalcemic effects develops within 1 or 2 days. Low calcium intake (C) is incorrect because it can result in increased hormone production and can accentuate bone disease. Calcium intake for this woman should be modest, but not low. Thiazides (D) are contraindicated because they can worsen hypercalcemia. When diuretics are used to control excessive or symptomatic hypercalcemia, furosemide is recommended, but never thiazides. Vitamin D analogs (B) are used in patients with hyperparathyroidism secondary to renal failure, in which case, the kidney can no longer add a hydroxyl group to vitamin D. This results in decreased calcium uptake from the GI tract In turn, PTH levels are elevated secondarily. Vitamin D analogs correct this problem. However, this patient has no problem with Vitamin D synthesis; giving her Vitamin D analogs would result in increased calcium absorption from the GI tract and would exacerbate the hypercalcemia.
-
Question 97 of 100
97. Question
1 pointsA 46 year old female has bone pain, myalgias, constipation, and a depressed mood. She has been hospitalized twice in the last year for urolithiasis. Labs reveal hypercalcemia and elevated PTH level. Her renal function is normal, and neck ultrasound reveals a parathyroid mass. Which laboratory abnormality would be expected in this patient?
Correct
Incorrect
Explanation:
The patient described is suffering from primary hyperthyroidism. The most common cause of primary hyperthyroidism is a parathyroid adenoma; parathyroid carcinoma and hyperplasia are much less common. This condition most commonly occurs in women over the age of 40 years as a solitary adenoma. Genetic syndromes such as MEN I, IIa, and IIb may also be associated with parathyroid adenomas, among other findings. Typical presenting complaints are those associated with hypercalcemia. These include recurrent urolithiasis, constipation, myalgias, bone pain, polyuria, fatigue, and psychologic changes. Patients may be asymptomatic at presentation, however, with the diagnosis made following discovery of an elevated calcium level on routine blood tests. Laboratory studies typically reveal an inappropriately elevated parathyroid hormone (PTH) level, hypercalcemia, hypophosphatemia, and elevated urine calcium. Chronically elevated PTH levels can cause defective bone mineralization and remodeling, leading to osteitis fibrosa cystica, a condition characterized by the formation of “brown tumors” in bone, particularly in the jaw. In cases in which the bone is significantly affected, the serum alkaline phosphatase is elevated, not decreased (E). Hypernatremia (A) typically occurs in the setting of excessive free water loss and is not associated with primary hyperparathyroidism. Hyperphosphatemia (B) and low levels of dihydroxy (24, 25(OH) 2) vitamin D occur in the setting of renal failure/renal osteodystrophy, one cause of secondary hyperparathyroidism. Hyponatremia (C) can occur in the syndrome of inappropriate antidiuretic hormone (SIADH), which is a paraneoplastic condition associated with small cell carcinoma of the lung. Another paraneoplastic syndrome associated with this malignancy is hypercalcemia caused by production of PTH related peptide by the malignant cells, resulting in hypercalcemia and low PTH levels.
-
Question 98 of 100
98. Question
1 pointsA man complains about constant burning, agonizing pain in his arm several months after sustaining a crushing injury to that arm, that does not respond to the analgesics. It is aggravated by slightest stimulation of the area, such as rubbing from the shirt sleeves. The arm is cold, cyanotic, and moist but not swollen. Pulses at the wrist are normal, and neurologic function of the three major nerves is intact. Which is appropriate to provide diagnostic confirmation for the nature of the problem and eventual therapy?
Correct
Incorrect
Explanation:
The description is that of causalgia, also known as reflex sympathetic dystrophy. If sympathetic block relieves the symptoms, permanent cure will be obtained with surgical sympathectomy. Venous occlusion (A) would produce swelling but not this kind of pain. Cervical ribs (B) can produce neurologic and vascular symptoms in the arm, but they are related to activity and position and do not have the nature described here. Normal pulses make arterial insufficiency (C) unlikely. Furthermore, there is no description of intermittent claudicating. Compartment syndrome (D) might have happened at the time of- injury, but if that were the case, it would be too late to do a fasciotomy. Compartment syndrome is characterized by excruciating pain with passive extension, tenderness to palpation, and a “tense” feel to the affected extremely. Neurologic function is lost initially, followed by pulses.
-
Question 99 of 100
99. Question
1 pointsA 66 year old male has not been able to void for the past 12 hours. He feels the need, but he cannot do it. For several years he has been getting up 4 or 5 times a night to urinate. It takes time to get the urinary stream going, and his stream is weak and often ended in a dribble. 2 days ago he began taking an antihistamine and a decongestant for a cold. Examination shows a pale man with a palpable, smooth, round mass arising from the pubis and reaching about halfway toward the umbilicus. The mass is dull to percussion, and pushing on it accentuates the feeling of needing to void. His temperature is 100.0°F. Rectal examination reveals a large, boggy, non tender prostate gland without nodules. The resident tries to catheterize his urethra and meets resistance. Which one would be the next step in management?
Correct
Incorrect
Explanation:
he history and rectal examination findings are classic for urinary retention caused by a benign prostatic hyperplasia (BPH). The use of a decongestant has led to stimulation of alpha adrenergic receptors, which have further closed the bladder neck, producing a big, palpable bladder The next step by general consensus is to catheterize the bladder with a Foley catheter. But when resistance or obstruction is encountered, it is necessary to do a percutaneous suprapubic cystostomy to relieve bladder distention. Dilation of the urethra is a valid option, although this should be done by a specially trained physician. Culture and sensitivity of prostatic secretions (A) would be a valid option if a bacterial prostatitis is suspected, which will be characterized by pain, fever spikes, and a very tender and hot prostate on rectal examination. This patient has a mild temperature elevation that is not due to infection. Morphine IV stat (B) may relieve the pain, but the same result can be achieved by relieving the urine retention. Furthermore, remember that opioid may increase the tone of the sphincters, an undesirable effect in this patient. To obtain a serum PSA level (C) is not indicated at this time. The case presentation is typical for BPH, and the patient is in distress due to the urinary retention. Relieve the pain and the obstruction first then consider ordering a PEA level. Trans-rectal ultrasound of the prostate (E) is indicated for early diagnosis of carcinoma of the prostate, as follow-up of abnormal digital rectal examinations, elevated PSA levels, evaluation of azoospermia, etc. It is not necessary in a patient with BPH facing acute urinary retention. The priority here is to relieve the patient´s bladder distention.
-
Question 100 of 100
100. Question
1 pointsA 62 year old woman has lower extremity weakness and worsening neck pain. Neck MRI shows chronic cervical stenosis with acute inflammatory changes consistent with edema. She is started on methylprednisolone. Next day, her symptoms are improved. That evening, she complains of sharp epigastric pain and vomits once. Routine labs are sent, and an AP CXR shows free air. The next step in the management of this patient is
Correct
Incorrect
Explanation:
The correct next step is to perform an urgent exploratory laparotomy this patient has pneumoperitoneum, and despite not knowing her clinical status (fever; hemodynamic, etc.), it needs to be surgically explored. Given the facts of her hospital admission, steroid use, and epigastric pain and vomiting, the diagnosis of a gastric ulcer should be obvious, and the presence of free air indicating perforation is an indication for urgent surgical exploration. Biopsies should also be taken to evaluate for Helicobacter pylori as well as a neoplastic process. Free air present on X-ray is almost ubiquitously an indication for exploration; further imaging with a CT scan (A) is not necessary this patient´s diagnosis should be recognized clinically and the decision to operate should be made on the basis of the plain X-ray findings. Concern for a peptic ulcer should be worked up with upper endoscopy (E), and H. pylori positive patients should be treated with triple therapy (D). However, that would be an option for the symptomatic but stable patient, not in the presence of known or suspected hollow bowel perforation. Endoscopy to evaluate the repair and triple therapy will likely be necessary for this patient in the future. NPO status and insertion of a nasogastric tube (C) will not help this patient in the long run. Gastric decompression and drainage are necessary to protect the repair, and a nasogastric tube must be placed under direct internal palpation in the operating room, but it is not the management of choice for this patient since it is not enough to repair the perforation.